+ All Categories
Home > Documents > ZSMU Immunology Mcqs for 5th yr MBBS

ZSMU Immunology Mcqs for 5th yr MBBS

Date post: 15-Oct-2014
Category:
Upload: drraghavender-reddy
View: 1,080 times
Download: 33 times
Share this document with a friend
Description:
Immunology mcqs from Zaporozhye state medical university,Ukraine,Eastren Europe
Popular Tags:
146

Click here to load reader

Transcript
Page 1: ZSMU Immunology Mcqs for 5th yr MBBS

ZAPORIZHZHIA STATE MEDICAL UNIVERSITY

CHAIR OF INTERNAL DISEASES №3

CLINICAL IMMUNOLOGY AND

ALLERGY

(in English)

COLLECTION OF TEST TASKS BY THE SYSTEM OF “KROK-2”

ZAPORIZHZHIA, 2010

Page 2: ZSMU Immunology Mcqs for 5th yr MBBS

«APPROVED»

Central Methodical Council ofZaporizhzhia State Medical University

Protocol № ____ from ____________ 2010

Authors:Sychov R.О. - Assistant of Chair of Internal Diseases № 3 of Zaporizhzhia

State Medical University, PhD

Dotsenko S.Y. – Head of Chair of Internal Diseases № 3 of Zaporizhzhia State

Medical University, Associate Professor, PhD

Kulynych О.V. – Associate Professor of Chair of Internal Diseases № 3 of

Zaporizhzhia State Medical University, PhD

The collection of tests in clinical immunology and allergy for self-

preparation workshops for students of the Medical Faculty of 5 and 6 training

courses learning in English. The collection of tests consists 464 tests and clinical

tasks in the system “KROK-2” with the standards of right answers and covers the

main sections of Clinical Immunology and Allergy (in English).

Reviewers:

- Professor of Chair of Family Medicine and Therapy FPE of Zaporizhzhia State

Medical University, Honored Scientist of Ukraine, Dr. of Sc., Deinega Volodymyr

Grygorovych.

- Associate Professor of Chair of Clinical Pharmacology, Pharmacy and

Pharmacotherapy of Zaporizhzhia State Medical University, PhD, Kras’ko Mykola

Petrovych.

2

Page 3: ZSMU Immunology Mcqs for 5th yr MBBS

ЗАПОРІЗЬКИЙ ДЕРЖАВНИЙ МЕДИЧНИЙ УНІВЕРСИТЕТ

КАФЕДРА ВНУТРІШНІХ ХВОРОБ №3

КЛІНІЧНА ІМУНОЛОГІЯ ТА

АЛЕРГОЛОГІЯ

(АНГЛІЙСЬКОЮ МОВОЮ)

ЗБІРНИК ТЕСТОВИХ ЗАВДАНЬ ЗА СИСТЕМОЮ «КРОК-2»

ЗАПОРІЖЖЯ, 2010 р.

3

Page 4: ZSMU Immunology Mcqs for 5th yr MBBS

«ЗАТВЕРДЖЕНО»

Центральною методичною радоюЗапорізького державногомедичного університету

Протокол № ____ від ____________ 2010 року

Автори:Сичов Р.О. - асистент кафедри внутрішніх хвороб № 3 Запорізького

державного медичного університету, к.мед.н.

Доценко С.Я. – завідувач кафедри внутрішніх хвороб № 3 Запорізького

державного медичного університету, доцент, к.мед.н.

Кулинич О.В. – доцент кафедри внутрішніх хвороб № 3 Запорізького

державного медичного університету, к.мед.н.

Збірник тестових завдань з клінічної імунології та алергології для

самостійної підготовки до практичних занять для студентів медичного

факультету 5 та 6 курсів навчання, які навчаються англійською мовою.

Збірник тестових завдань складений з 464 тестових завдань та клінічних

задач за системою «КРОК-2» з еталонами вірних відповідей з основних

розділів клінічної імунології та алергології (англійською мовою).

Рецензенти:

- Професор кафедри сімейної медицини і терапії ФПО Запорізького

державного медичного університету, заслужений діяч науки і техніки

України, доктор медичних наук, Дейнега Володимир Григорович.

- Доцент кафедри клінічної фармакології, фармації і фармакотерапії

Запорізького державного медичного університету, кандидат медичних наук

Красько Микола Петрович.

4

Page 5: ZSMU Immunology Mcqs for 5th yr MBBS

1. The most scientific method to identify which triggers may be causing my nasal allergies is to:

A. Track the time of year when my symptoms occurB. Have an allergist perform a skin testC. Have an allergist run a blood testD. There is no good way to know what I'm allergic toE. B and/or C

2. Which is the most effective treatment for nasal allergies?A. Over-the-counter antihistaminesB. Prescription antihistaminesC. Antihistamine decongestant combination pillsD. Nasal corticosteroid spraysE. Oral leukotriene receptor antagonists

3. Which one of the following is least likely to trigger asthma?A. Strenuous exerciseB. A common coldC. Reading the newspaperD. Cat danderE. Tobacco smoke

4. Which of the following factors can pre-dispose you to having allergies?A. A family history of allergiesB. Environmental ConditionsC. Number and type of ExposuresD. Emotional factorsE. All of the above

5. Hypersensitivity of which system of your body creates an allergic response?A. SkinB. Immune SystemC. Circulatory SystemD. Pulmonary SystemE. Digestive System

6. Which of the following treatments can be used to reduce allergic reactions to unavoidable substances?

A. Milk thistle three times dailyB. Mist inhalation therapyC. Regularly wearing a face maskD. Desensitization (Allergy shots)E. Regularly wearing a garlic necklace

7. Which of the following is the most commonly prescribed treatment for hives?

5

Page 6: ZSMU Immunology Mcqs for 5th yr MBBS

A. Oral antihistaminesB. Oral corticosteroids such as prednisoneC. Epinephrine (adrenaline)D. AspirinE. Immunotherapy

8. Which of the following does NOT reduce or delay allergies in children?A. Reducing dust-collecting itemsB. Limiting exposure early in life to indoor furry petsC. Avoiding smoking in and around the houseD. Exclusively wearing cotton clothesE. All actions adequately reduce allergies

9. Which of these body systems causes allergic reactions?A. LymphB. ImmuneC. NervousD. AutonomicE. Circulatory

10. An allergen is anything that triggers an allergic or hypersensitive response. Which of these could be an allergen?

A. DustB. FoodC. Nickel jewelryD. All of the aboveE. None of the above

11. What does the body release to combat allergens?A. PlasmaB. EpinephrineC. HistamineD. Red Blood cellsE. Cortisone

12. The most severe form of allergic reaction is called anaphylaxis. What happens?A. Blood pressure dropsB. Breathing becomes difficultC. Runny nose developsD. A and BE. B and C

13. Which of these symptoms may indicate a food allergy?A. Skin rashB. Runny nose

6

Page 7: ZSMU Immunology Mcqs for 5th yr MBBS

C. DiarrheaD. NauseaE. Wheezing

14. Which of these foods are most likely to trigger an allergy?A. ShellfishB. WheatC. CeleryD. A and BE. B and C

15. More Americans are developing a sensitivity to the plant proteins in latex materials or the chemical additives used in manufacturing the materials. Who's most at risk of developing a latex allergy?

A. Patients who undergo numerous medical proceduresB. Health care workersC. ChildrenD. A and BE. B and C

16. Pediatricians who treat children with egg allergies are cautious about using certain vaccines because they may be grown in an egg medium. Which of these vaccines warrant caution?

A. MMRB. FluC. PolioD. A and BE. B and C

17. Dust mites trigger indoor respiratory allergies. Where are you most likely to find them in the home?

A. CarpetB. BedsC. DrapesD. All of the aboveE. None of the above

18. How long ago was asthma identified as an ailment?A. 1,000 years ago B. 500 years ago C. 100 years ago D. 50 years agoE. 100,000 years ago

19. What are the symptoms of asthma?

7

Page 8: ZSMU Immunology Mcqs for 5th yr MBBS

A. Tightness in the chestB. Wheezing C. Sneezing D. A and BE. B and C

20. Which of these can trigger an asthma attack?A. Exercise B. Aspirin C. Laughing D. All of the aboveE. None of the above

21. Which of these allergens are most likely to induce asthma?A. Ragweed B. Cockroaches C. Feathers D. B and CE. A and B

22. Which of these foods should children with asthma avoid?A. Dairy products B. All nuts C. Foods with preservatives D. None of the aboveE. All of the above

23. Which of these, if eaten on a weekly basis, can help reduce chronic coughing and wheezing in asthmatic children?

A. Broccoli B. Oranges C. Apples D. FishE. Peanut

24. Which sports are best for asthmatics?A. Baseball B. Swimming C. Sprinting D. All of the aboveE. Sport is dangerous for asthmatics

25. What can you do to reduce exposure to outdoor asthma triggers?A. Exercise outdoors on high ozone days B. Schedule camping trips in May, June or August through October

8

Page 9: ZSMU Immunology Mcqs for 5th yr MBBS

C. Pursue outdoor sports in cold weather D. None of the aboveE. All of the above

26. Which of these alternative health treatments have reduced symptoms in children?

A. Nightly massages B. Evening primrose herb C. Yoga D. AromatherapyE. All of the above

27. What should you do if pet hair triggers your asthma?A. Bathe the pet weekly B. Keep the pet outside if possible C. Own a dog, not a cat D. All of the aboveE. None of the above

28. People who have atopic dermatitis also may have:A. Asthma B. Allergies C. Acne D. A and BE. B and C

29. Which group of people is more likely to develop atopic dermatitis?A. Infants and young children B. Teenagers C. Adults 20 to 49 D. Older adultsE. All of the above

30. One characteristic of atopic dermatitis is:A. It affects the face more than the rest of the body B. It can leave pockmarks on the skin C. It cycles through periods of flares and remissions D. It is worse in autumnE. There is no correct answer

31. Atopic dermatitis is also often called:A. Acne B. Eczema C. Psoriasis D. Pimples

9

Page 10: ZSMU Immunology Mcqs for 5th yr MBBS

E. Hives

32. In the past, doctors thought which of these caused atopic dermatitis?A. Too much sun B. An emotional disorder C. Food allergies D. Dust mite allergyE. None of the above

33. Which is a symptom of atopic dermatitis?A. Blisters on the palms of hands and soles of feet B. Itchy, inflamed skin C. Scaly patches of skin on the scalp D. Coin-shaped patches of irritated skin on the arms and lower legsE. All answers are correct

34. Some people with atopic dermatitis develop:A. Thick, leathery skin B. Red, scaling skin C. Small raised bumps on the skin D. All of the aboveE. None of the above

35. The skin of a person with atopic dermatitis is more susceptible to:A. Skin infections B. Warts C. Herpes simplex D. All of the aboveE. None of the above

36. Match the immune deficiency in Wiskott-Aldrich syndrome with the appropriate lab test:

A. Quantitative immunoglobulinsB. CD11 by flow cytometryC. NBT test (nitroblue tetrazolium test, oxidated burst) D. Serum calciumE. Platelet count and morphology

37. Match the immune deficiency in Di George anomaly with the appropriate lab test:

A. Quantitative immunoglobulinsB. CD11 by flow cytometryC. NBT test (nitroblue tetrazolium test, oxidated burst) D. Serum calciumE. Platelet count and morphology

10

Page 11: ZSMU Immunology Mcqs for 5th yr MBBS

38. Match the immune deficiency in Bruton’s disease with the appropriate lab test:A. Quantitative immunoglobulinsB. CD11 by flow cytometryC. NBT test (nitroblue tetrazolium test, oxidated burst) D. Serum calciumE. Platelet count and morphology

39. Match the immune deficiency in Leukocyte adhesion defect with the appropriate lab test:

A. Quantitative immunoglobulinsB. CD11 by flow cytometryC. NBT test (nitroblue tetrazolium test, oxidated burst) D. Serum calciumE. Platelet count and morphology

40. Match the immune deficiency in Chronic Granulomatous disease with the appropriate lab test:

A. Quantitative immunoglobulinsB. CD11 by flow cytometryC. NBT test (nitroblue tetrazolium test, oxidated burst) D. Serum calciumE. Platelet count and morphology

41. State one from airborne substances listed below is Perennial allergen:A. Dust mitesB. SagebrushC. RagweedD. PerfumeE. Cigarette smoke

42. State one from airborne substances listed below is Non-IgE mediated irritant:A. Dust mitesB. Cat danderC. RagweedD. SagebrushE. Cigarette smoke

43. State one from airborne substances listed below is Seasonal allergen:A. Dust mitesB. Cat danderC. RagweedD. PerfumeE. Cigarette smoke

11

Page 12: ZSMU Immunology Mcqs for 5th yr MBBS

44. In June, a 24-year-old female presents with angioedema of the face, including eyelids and lips. She has had two previous similar episodes. Her father has also had recurrent angiodema of the face and extremities. Neither has associated pruritis with the angioedema. Which diagnosis is suggested?

A. Hereditary angioedemaB. Food allergyC. Enterovirus infectionD. Idiopathic angioedemaE. An ACE inhibitor was prescribed by her internist for hypertension

45. In June, a 24-year-old female presents with angioedema of the face, including eyelids and lips. She has had two previous similar episodes. She always has associated gas and diarrhea with the angioedema. She was previously skin tested and found to have sensitivities to milk, ragweed and molds. Which diagnosis is suggested?

A. Hereditary angioedemaB. Food allergyC. Enterovirus infectionD. Idiopathic angioedemaE. An ACE inhibitor was prescribed by her internist for hypertension

46. In June, a 24-year-old female presents with angioedema of the face, including eyelids and lips. She has had two previous similar episodes. This episode, like the previous ones, occurred during the summer and was accompanied by diarrhea, low grade fever once and classical urticarial lesions.

A. Hereditary angioedemaB. Food allergyC. Enterovirus infectionD. Idiopathic angioedemaE. An ACE inhibitor was prescribed by her internist for hypertension

47. Hereditary angioedema is:A. Inherited as an autosomal recessive disorderB. Usually associated with urticariaC. Usually responsive to antihistaminesD. Can be managed with Danazol (synthetic androgenic steroid)E. All of the above are correct

48. Which of the following adverse drug reactions is believed to be due to IgE-mediated mechanisms?

A. Aspirin induced anaphylaxisB. Reaction to radiocontrast dyeC. Steven-Johnson’s Syndrome after a course of trimethoprim/sulfaD. Wheezing, urticaria after administration of penicillinE. None of the above

12

Page 13: ZSMU Immunology Mcqs for 5th yr MBBS

49. Which of the following food allergies is most likely outgrown?A. MilkB. PeanutC. WalnutsD. ShellfishE. All of the abova

50. Indications for immunotherapy for hymenoptera allergy:A. Children and adults with a history of a life-threatening reaction to a

hymenoptera stingB. People who have a history of a large local skin reaction that gets worse

with each stingC. Children with a history of generalized urticaria after a stingD. Anyone with a family history of an allergic reaction to a hymenoptera stingE. Patients with allergic rhinitis

51. Eczema in an infant most commonly occurs in/on the:A. Antecubitutal and Popliteal fossaeB. Perineal regionC. Scalp and flexural areasD. Extensor surface of arm and legsE. All of the above

52. As part of the work up of adult asthma, which of the following reproducible, pre and post bronchodilator pulmonary function test results would be interpreted as showing significant reversibility?

A. a 15% increase in FEF 25-75%B. a 15% increase in FEV1 (425 ml improvement)C. a 20% increase in FEV1 (180 ml improvement)D. a 15% increase in FEV1/FVC ratioE. a 15% increase in FVC

53. All of the following medications can affect late phase (including by prophylaxis of the immediate allergic response) response in asthma except:

A. CorticosteroidsB. Leukotriene modifiersC. Cromolyn sodiumD. AlbuterolE. There is no such medication in the list

54. Drug of choice for uncomplicated acute urticaria is:A. CorticosteroidB. Antihistamines (H2 type)C. Antihistamines (H1 type)

13

Page 14: ZSMU Immunology Mcqs for 5th yr MBBS

D. Subcutaneous terbutalineE. All of the above

55. Reasonable recommendations for a patient with moderate persistent asthma poorly controlled on low dose inhaled corticosteroids include all except:

A. Add salmeterol inhaler BIDB. Double the dose of inhaled corticosteroidC. Add a leukotriene modifierD. Add nebulized cromolyn BIDE. All answers are correct

56. Which of the following is a clear indication for penicillin allergy skin testing:A. A 40-year-old patient with a history of anaphylaxis after ampicillin ten

years ago. The patient now has endocarditis. Culture is positive for a streptococcal organism sensitive only to penicillin.

B. A 26-year-old patient with a history of Steven-Johnson’s Syndrome after receiving amoxicillin.

C. A 65-year-old ICU patient receiving penicillin and vancomycin with flushing and hypotension.

D. A 39-year-old female who, eleven days after taking penicillin for a strep pharyngitis, develops angioedema, arthralgias, urticaria and a low grade fever.

E. All of the above.

57. The major advantage of second generation antihistamines is:A. They do cross the blood brain barrier and help patients to relaxB. They are very long actingC. They cause decongestion as well as decreasing rhinorrheaD. They minimally cross the blood brain barrier and have a decreased sedation

effectE. They decrease appetite

58. Which of the following types of infection are increased in patients with humoral or B-cell immune deficiencies?

A. SinusitisB. Otitis mediaC. PneumoniaD. MeningitisE. All of the above

59. The most common cause of chronic urticaria, especially in adults, is:A. Food allergyB. Connective tissue diseaseC. Drug allergyD. Idiopathic

14

Page 15: ZSMU Immunology Mcqs for 5th yr MBBS

E. Viral infections

60. Congenital absence of the late complement components (C5, C6, C7, C8) is most often associated with:

A. Viral infectionsB. Recurrent Neisserial infectionsC. Lupus-like diseaseD. CandidiasisE. Delayed separation of the umbilical cord

61. Congenital complement deficiencies are best screened by which of the following laboratory tests:

A. Quantitative immunoglobulinsB. NBTC. Total CH50D. C3, C4 and properdinE. Flow Cytometry

62. All of the following except one can be useful in managing atopic dermatitis:A. Skin lubrication/moisturizationB. Controlling pruitus to reduce scratchingC. Immunotherapy (allergy injections)D. Avoiding specific food allergy triggersE. Topical therapy with steroid or non-steroid immune modulating agent

(pimecolimus, tacrolimus)

63. All of the following regarding aspirin intolerance are true except:A. Patients with asthma are at no greater risk for aspirin intolerance than the

general populationB. A proposed mechanism of aspirin intolerance involves aspirin’s effect on

arachidonic acid metabolismC. There is cross reactivity between aspirin and NSAID intoleranceD. Aspirin desensitization may be of benefit in a patient with aspirin

intolerance who requires the drugE. Leukotriene modifiers are particularly beneficial in most patients

64. Which of the following tests would be most useful in helping to diagnose asthma:

A. Diffusing capacityB. Pre and post bronchodilator FEV1C. Arterial blood gasesD. Pulmonary functions pre- and post-methacholine challengeE. B and D

15

Page 16: ZSMU Immunology Mcqs for 5th yr MBBS

65. Conditions which may worsen asthma include (Identify the one incorrect choice):

A. Diabetes MellitusB. SinusitisC. PregnancyD. Cardiac diseaseE. Gastro-esophageal reflux

66. Which of the following is the most common underlying cause of asthma (Choose the one best answer):

A. Beta Blocker sensitivityB. Aspirin sensitivityC. BronchitisD. SinusitisE. Inhalant allergies

67. Inhaled corticosteroids (Choose the one incorrect answer):A. Can be useful in patients who have persistent asthmaB. Can be useful in patients who require rescue inhaler 1-2 times per weekC. Can be used in 6-12 year-old children with asthmaD. Acutely increase peak flow rate, a measure of large airway functionE. Reduce airway hyper-reactivity

68. A 40-year-old male presents to the Emergency Room complaining of hives, wheezing and difficulty breathing following an insect sting. Pulse is 120 beats/min. BP is 100/56. Immediate therapy should include which one of the following:

A. Diphenhydramine 50 mg IVB. Methylprednisolone 100 mg IVC. Epinephrine 0.3 ml (1:1000 aqueous) sub Q or IMD. Cholorpheniramine 8 mg orallyE. None of the above

69. A 40-year-old male presents to the Emergency Room complaining of hives, wheezing and difficulty breathing following an insect sting. Pulse is 120 beats/min. BP is 100/56. After Epinephrine is administered and quick assessment of the patient is done, you determine that the patient is moving air adequately and is not in shock. The next medication to give the patient immediately is:

A. Diphenhydramine 50 mg IVB. Methylprednisolone 100 mg IVC. Epinephrine 0.3 ml (1:1000 aqueous) sub Q or IMD. Cholorpheniramine 8 mg orallyE. None of the above

70. Which of the following symptoms is least common in patients with sinusitis?A. Nasal congestion

16

Page 17: ZSMU Immunology Mcqs for 5th yr MBBS

B. CoughC. FeverD. “Post-nasal” dripE. Mucopurulent rhinorrhea

71. Which of the following would not suggest a FISH (Fluorescence In Situ Hybridization) test to identify DiGeorge syndrome?

A. Recurrent otitis media in a 3-year-oldB. Neonatal tetanyC. Pervasive developmental delay in a 6-year-old femaleD. Recurrent/persistent otitis and diarrhea in a 6-month-oldE. None of the above

72. Hereditary angioedema (C1 inhibitor deficiency) is characterized by all of the following except:

A. Frequent abdominal painB. Urticaria or hivesC. Low C4 levels during and between attacksD. Family history of swellingE. Absence of symptoms before puberty

73. Common causes of acute urticaria include all of the following except:A. Food allergyB. Drug allergyC. Viral infectionD. Inhalant allergyE. Hymenoptera stings

74. The two most common superinfections in atopic dermatitis are due to:A. Staph aureus and Herpes simplexB. Staph epidermidis and cutaneous viral infectionsC. Staph aureus and CandidaD. Staph epidermidis and CandidaE. Acinetobacter and Herpes simplex

75. The most common cause of contact dermatitis in general is:A. LatexB. Poison ivy, poison oak and poison sumacC. NickelD. Dark-colored pigments in shoes and clothingE. None of the above

76. A 2-year-old with wheezing, persistent cough, a history of vomiting with cough, as well as hives and angioedema and increased cough after milk products comes to your office for evaluation. He drinks soy and grandma is not feeding him

17

Page 18: ZSMU Immunology Mcqs for 5th yr MBBS

any milk products. He has been treated with prn albuterol nebulizer treatments with benefit in the past. On exam he has frequent congested cough, p = 112, r =26, shiners and Dennies lines. On chest exam he has transmitted upper airway rhonchi and coarse breath sounds but no wheezes or rales. Nasal exam reveals swollen pale nasal mucosa without any visible discharge. CXR shows increased bronchovascular markings without infiltrate but with hyperexpansion. What diagnostic lab information do you want to obtain?

A. CBC and diff, IgE and IgG RAST to milk and soyB. CBC and diff, total IgE, IgG RAST to milkC. CBC and diff, total IgE, and skin or RAST testing to milk, casein and soyD. CBC, milk precipitins, IgE RAST to milk and caseinE. B and D

77. The one true statement about anaphylaxis isA. There are always skin findings—rash, angioedema, etc.B. People with recurrent anaphylaxis with no identifiable cause (idiopathic)

are the most likely to carry adrenalinC. After adrenalin, the most important medication to administer immediately

is steroidsD. After adrenalin, the most important medication to administer promptly is

an H2 blockerE. The most likely patient to die of anaphylaxis has underlying asthma

78. A 5-year-old boy lives in a smoking household and has a history of RSV bronchiolitis in infancy. He has been diagnosed with bronchitis at least once, and sometimes twice, every winter. He has sneezing and allergic salute in spring and fall. When he runs in kindergarten he often coughs but has no obvious shortness of breath. He wakes at night coughing only when he has bronchitis. He is unable to do full pulmonary functions. The historical information most suggestive of asthma in this child is:

A. Cough at night when illB. His allergic symptomsC. RSV bronchiolitis in infancyD. Cough with exerciseE. He has been living in a smoking household

79. A 45-year-old woman with a strong family history of both allergies and glaucoma presents with frequent red eyes that sometimes itch, sometimes feel dry and sometimes water. On exam, bulbar conjunctivae are injected, palpebral conjunctiva are normal in color. Nasal mucosa is moderately swollen and pale. Which of the following is the most likely diagnosis?

A. Vernal conjunctivitisB. Allergic conjunctivitisC. GlaucomaD. Dryness from hormone replacement

18

Page 19: ZSMU Immunology Mcqs for 5th yr MBBS

E. Behcet’s disease

80. An 8-year-old male has asthma symptoms about three times per month requiring the use of an albuterol inhaler, despite adequate inhaled steroid treatment. These episodes typically occur at night. The child is not allergic to his guinea pig or dust mites, and there is no smoking in the home. He has no problems with exercise and rarely needs albuterol during the day except when he has an upper respiratory tract viral infection. His FEV1 and PEFR are both >80% predicted. Based on this history, the most likely trigger for his nighttime asthma is:

A. Allergic rhinitisB. Obstructive sleep apneaC. Gastro-esophageal reflux disease (GERD)D. Drop in endrogenous corticosteroid levels at nightE. None of the above

81. A 10-year-old girl has had asthma for 18 months. She typically complains of asthma symptoms 3 times per week on the average, with daily symptoms when she has a URIs. She has never been hospitalized for asthma but has had 2 ED visits for wheezing with URIs. She complains of cough and shortness of breath with strenuous exercise. Her FEV1 and PEFR are both >80% predicted. Her treatment to date has been an albuterol inhaler prn. At this point, the one clearly inadequate therapy for this patient is:

A. Start a leukotriene antagonist daily, albuterol inhaler prn and before exercise

B. Start an inhaled corticosteroid at low to moderate dose, continue albuterol prn and before exercise

C. Start the patient on a combined controller inhaler with inhaled steroid and long acting bronchodilator

D. Start the patient on an inhaled steroid and leukotriene antagonist, and albuterol inhaler before exercise and prn

E. All therapy strategies listed are adequate for this patient

82. A 52-year-old male with known asthma for 27 years is seen in your office for a follow up of his asthma. He states his asthma has been in good control on his low dose of inhaled corticosteroid. But when asked about albuterol use, he says he uses it 2-3 times daily. When asked about nighttime awakening due to asthma, he states he awakes because of asthma 3 times per month. He also tells you his asthma has been in good control and that he has not missed a day of work. A PFT indicates he has an FEV1 of 81% predicted. You would characterize his asthma as:

A. Mild intermittentB. Mild persistentC. Moderate persistentD. Severe persistentE. Well controlled asthma

19

Page 20: ZSMU Immunology Mcqs for 5th yr MBBS

83. In treating allergic rhinitis, which of the following medication or medications control the symptoms of congestion, rhinorrhea and itching.

A. AntihistamineB. AnticholinergicC. DecongestantD. Nasal corticosteroidE. Anti-leukotriene receptor antagonist

84. An 18-year-old teenager with a history of asthma as a young child, which she “outgrew” by the time she was in first grade, presents to her primary care physician in November 4 months pregnant, with wheezing and cough for the last month. On questioning, she admits to ragweed hayfever and frequent heartburn. On exam, she is coughing at frequent intervals. Chest is clear to auscultation with fair breath sounds throughout. There is slightly prolonged expiratory phase. What is the most appropriate treatment for this patient:

A. Prn albuterol inhaler, loratadine and calcium carbonate antacid.B. Prn albuterol inhaler, beclomethasone inhaled corticosteroid and ranitidine.C. Prn albuterol inhaler, budesonide inhaled corticosteroid, ranitidine and oral

steroid burst.D. Prn albuterol inhaler, oral steroid burst and proton pump inhibitor.E. Prn albuterol inhaler, course of erythromycin for possible mycoplasma

infection, calcium carbonate antacid.

85. Complement activation is a part of which type of hypersensitivity reaction?A. Type IB. Type II and Type IIIC. Type IVD. Type III and IVE. Type I and IV

86. After playing in the bushes during a camping trip, a 7-year-old girl complains of intense itching & blistering of the hands, arms, & legs. What is the most likely condition & what type of hypersensitivity reaction is she displaying?

A. Hives; Type I reactionB. Hives; Type IV reactionC. Contact Dermatitis; Type IV reactionD. Contact Dermatitis; Type I reactionE. SLE; Type V reaction

87. Skin testing is useful in the diagnosis of which type of hypersensitivity reaction?

A. Type IB. Type II and Type IIIC. Type IVD. Type III and IV

20

Page 21: ZSMU Immunology Mcqs for 5th yr MBBS

E. Type I and IV

88. Rh disease & Goodpasture's syndrome are which type of hypersensitivity reaction?

A. Type IB. Type IIC. Type IIID. Type IVE. Type V

89. Megakaryocytes are minimally immunoresponsive. What type of cell do megakaryocytes give rise to?

A. RBCB. PlateletsC. LeukocytesD. MonocytesE. Mast cells

90. A latent, measles-like viral infection &, presumably, a defect in cellular immunity is associated with which of the following diseases?

A. Hereditary angioedemaB. Systemic lupus erythematous (SLE)C. Subacute sclerosing panencephalitis (SSPE)D. DiGeorge syndromeE. Bruton disease

91. What host cell membrane structures enable the attachment of a virus like human immunodeficiency virus (HIV)?

A. Ion channelsB. Nuclear poresC. RibosomesD. ReceptorsE. All of the above

92. Name the process a cell such as a neutrophil or a macrophage uses to ingest (eat) its prey.

A. HalitosisB. ChemotaxisC. BotulismD. PhagocytosisE. Pinocytosis

93. In HIV infection, reverse transcription describes which of the following?A. Converting viral DNA into RNA

21

Page 22: ZSMU Immunology Mcqs for 5th yr MBBS

B. Converting viral RNA into DNAC. Converting proteins into viral RNAD. Both A and BE. Both B and C

94. Which of these produces and secretes antibodies in the body?A. BacteriaB. Plasma CellC. Red blood cellD. VirusE. Both A and D

95. What's a specific term for a bacterial or other foreign protein that initiates antibody production by the body?

A. PeptideB. MHC II moleculeC. ComplementD. AntigenE. Allergen

96. What product of the immune system attaches to bacteria, making them easier to eat by white blood cells?

A. HemoglobinB. AntibodyC. AntigenD. Both A and BE. Both B and C

97. What is an important mechanism white blood cells use to kill bacteria, fungi and other invading pathogens?

A. AsphyxiationB. Oxidative activityC. FrightD. DrowningE. All of the above

98. What is the term applied to white blood cells squeezing between endothelial cells lining the blood vessel to reach the site of an infection?

A. DiapedesisB. ChemotaxisC. PhagocytosisD. EnucleationE. All of the above

99.What HIV protein helps insert the HIV provirus into the host's DNA?

22

Page 23: ZSMU Immunology Mcqs for 5th yr MBBS

A. TranscriptaseB. IntegraseC. ProteaseD. DismutaseE. Oxydase

100. What main characteristic of HIV makes the virus difficult to treat with a single drug?

A. Its ability to destroy drugsB. Impenetrable capsuleC. Its ability to mutateD. Its environmental durabilityE. RNA containing

101. Which organ is NOT part of the immune system?A. ThymusB. BrainC. TonsilsD. SpleenE. B and C are correct

102. Which part of the body does NOT contain lymph nodes?A. KneesB. ArmpitsC. NeckD. FeetE. A and D are correct

103. What type of immunity is developed through exposure to a disease?A. Acquired passive immunityB. Passive artificial immunityC. Active artificial immunityD. Acquired active immunityE. A and C are correct

104. What is a substance that can cause a person to become sensitive to, and produce antibodies against it?

A. ComplementB. ThymosinC. HumorD. AntigenE. Epinephrine

105. What common allergic disorder was named for the illness first described in those exposed to the farmlands of England?

23

Page 24: ZSMU Immunology Mcqs for 5th yr MBBS

A. Systemic lupus erythematosusB. Hay feverC. DiabetesD. AsthmaE. Anemia

106. Which of these is an antiviral agent?A. AveloxB. ZoviraxC. DiflucanD. AmoxilE. Benadryl

107. Which of the following tests would most likely be used for suspicion of HIV?A. Patch testB. Scratch testC. ELISAD. Complete blood countE. Lymphangiography

108. What is the largest lymphoid structure?A. BrainB. ThymusC. Lymph nodesD. SpleenE. Tonsils

109. The tough, water insoluble protein present in the outer portion of the epidermis of the skin is:

A. EpiderminB. CaroteneC. KeratinD. ImpenetrinE. Other substance

110. Which one of the following is not a nonspecific defense mechanism? A. SkinB. Mucous membraneC. TearsD. Cell mediated immunityE. All of the above

111. Which of the following occurs in the acute, nonspecific inflammatory response?

A. Margination and chemotaxix of neutrophils

24

Page 25: ZSMU Immunology Mcqs for 5th yr MBBS

B. Vasodilation of small blood vesselsC. Phagocytosis by macrophagesD. Increased permiability of small blood vesselsE. All of the above

112. Which of the following does not occur as part of the acute, nonspecific inflammatory response?

A. Aggregation of neutrophils at the site of injuryB. Slowing of blood flow in the small blood vesselsC. Vasoconstriction of the small blood vesselsD. All of the aboveE. None of the above

113. The process whereby leucocytes are displaced to the periphery of the blood stream and start to adhere to the blood vessel walls is known as?

A. AggregationB. DiapedesisC. PhagocytosisD. MarginationE. Opsonisation

114. The movement of leucocytes through tissues towards the site of injury is an example of:

A. MarginationB. ChemotaxisC. ApoptosisD. PhagocytosisE. Mitosis

115. During acute, nonspecific inflammation, the first leucocytes to arrive at the site of tissue damage are:

A. MonocytesB. EosinophilsC. MacrophagesD. LymphocytesE. Neutrophils

116. During the later stages of an acute, nonspecific inflammatory response, the kinds of leucocytes that predominate at the site of tissue injury are usually:

A. Lymphocytes and neutrophilsB. Macrophages and monocytesC. Basophils and megakaryocytesD. T and B lymphocytesE. Basophils and eosinophils

25

Page 26: ZSMU Immunology Mcqs for 5th yr MBBS

117. Leucocytes expressing CD-4 receptors are:A. Plasma cellsB. B-lymphocytesC. T-cytotoxic cellsD. T-helper cellsE. T-supressor cells

118. Leucocytes expressing CD-8 receptors are:A. Plasma cellsB. B-lymphocytesC. T-cytotoxic cellsD. T-helper cellsE. T-supressor cells

119. The B-lymphocyte antigen receptor is:A. Ig MB. Ig GC. Ig AD. Ig DE. Ig E

120. Which of the following cells contain or release histamine? A. Mast cellsB. BasophilsC. PlateletsD. All of the aboveE. None of the above

121. Which of the following statements is not true of the complement system? A. They act as chemotactic agentsB. They form pores in bacterial cell wallsC. They coat the surface of bacteriaD. Inactive forms normally circulate in the plasmaE. All of the above

122. Which of the following does not contribute to the vasodilation of small blood vessels during a Localised acute non-specific inflammatory response?

A. HistamineB. ProstaglandinsC. KininsD. Substance PE. All of the above contribute

123. Which of the following is a symptom of inflammation? A. Hyperaemia

26

Page 27: ZSMU Immunology Mcqs for 5th yr MBBS

B. SwellingC. PainD. Increased temperatureE. All of the above

124. Which of the following is a common symptom of inflammation? A. ParalysisB. ShockC. NauseaD. All of the aboveE. None of the above

125. Which of the following represents a normal white cell blood count? A. 400 - 500 cells per cubic mmB. 1000 - 2000 cells per cubic mmC. 4000 - 8000 cells per cubic mmD. 20,000 - 30,000 cells per cubic mmE. Far more than the above

126. Bacterial invasions may cause the white cell count to be? A. 400 - 500 cells per cubic mmB. 1000 - 2000 cells per cubic mmC. 4000 - 8000 cells per cubic mmD. 20,000 - 30,000 cells per cubic mmE. Far less than the above

127. Causes of an elevated white cell count include: A. Bacterial infectionB. Viral infectionC. Severe physical or emotional stressD. Acute or chronic leukemiaE. All of the above

128. Which of the following is not a specific immune response? A. Acute inflammationB. Cell mediated immunityC. Humoral immunityD. Antigen presentationE. All of the above

129. Which of the following cell types is not involved in an antibody mediated response?

A. B lymphocytesB. Plasma cellsC. T-cytotoxic cells

27

Page 28: ZSMU Immunology Mcqs for 5th yr MBBS

D. T-helper cellsE. All of the above

130. Which of the following cell types is involved in a cell mediated response? A. B lymphocytesB. Plasma cellsC. T-cytotoxic cellsD. T-helper cellsE. All of the above

131. The cells that produce and secrete antibodies are:A. T cellsB. B cellsC. Plasma cellsD. Dendritic cellsE. All of the above

132. A person does not normally produce antibodies or activated T cells to self antigen due to the presence of:

A. HaptensB. ImmunoglobulisC. Human leucocyte antigenD. Non-self antigenE. All of the above

133. Which region of an antibody molecule that enables it to recognise and bind to a specific antigen?

A. The light chainsB. The heavy chainsC. The constant regionD. The variable regionE. Complement binding region

134. Within what time phagocytic white cells (leukocytes, e.g. macrophages) congregate when foreign organisms get through a cut in the skin?

A. MicrosecondsB. SecondsC. MinutesD. HoursE. Days

135. Which of the following mediates an early response to viral infections by the innate immune system?

A. Complement components

28

Page 29: ZSMU Immunology Mcqs for 5th yr MBBS

B. VaccinesC. T and B lymphocytesD. CytokinesE. Interferons

136. Which of the following is a messenger that mediates the connection between the innate and adaptive immune systems?

A. Complement componentsB. VaccinesC. T and B lymphocytesD. CytokinesE. Interferons

137. Which of the following immune system components would NOT recognize a macromolecule epitope (binding site)?

A. PhagocyteB. T lymphocyteC. B lymphocyteD. AntibodyE. Plasma cell

138. Which of the following is a large genomic region or gene family found in most vertebrates, playing an important role in immunity?

A. Antigen-recognition moleculesB. Major histocompatibility complexes (MHCs)C. Human leukocyte antigens (HLAs)D. ImmunoglobulinE. Epitopes

139. How are B-cell and T-cell antigen receptors distributed?A. By size (steric hindrance).B. By molecular weightC. By activation (clonal)D. By location in the bodyE. Equally (same number of each receptor)

140. A medical student acquired hepatitis B through a needle-stick injury. The student had not received their vaccinations for this virus and eventually experienced liver problems. One of the clinical signs of this disease is jaundice, which turns the skin and sclera what color?

A. BlackB. RedC. WhiteD. YellowE. Blue

29

Page 30: ZSMU Immunology Mcqs for 5th yr MBBS

141. Adaptive immune system response typically takes how long/A. MicrosecondsB. SecondsC. MinutesD. HoursE. Days

142. Which of the following is NOT true when comparing innate to adaptive immunity?

A. Innate responds quickly and adaptive responds slowlyB. Innate has few pathogen (non-self) recognition mechanisms and adaptive

has manyC. Innate has immunologic memory and adaptive does notD. Innate does not show response improvements over time and adaptive doesE. Innate response is non-specific and adaptive is very specific

143. Phagocytes ingest particular matter into cells for degradation. Which of the following is NOT considered a phagocyte?

A. MacrophageB. NeutrophilC. EosinophilD. BasophilE. Lymphocyte

144. Which of the following components of the innate immune system involves recognition molecules such as mannan-binding lectin (MBL) for bacteria with mannose on the surface?

A. Natural killer (NK) cellsB. Complement systemC. Oxygen dependent and independent killingD. InterferonsE. Acute phase proteins

145. Which of the following components of the innate immune system involves cytokines produced by macrophages, which are produced during infection?

A. Natural killer (NK) cellsB. Complement systemC. Oxygen dependent and independent killingD. InterferonsE. Acute phase proteins

146. Which of the following components of the innate immune system involves the release of histamine?

A. Neutrophil

30

Page 31: ZSMU Immunology Mcqs for 5th yr MBBS

B. EosinophilC. MacrophageD. Tissue mast cellE. Natural killer cell

147. Which of the following components of the adaptive immune system secretes immunoglobulin (Ig)?

A. Activated B cell (plasma cell)B. CD4+ activated T cellC. CD8+ cytotoxic T cell (CTL)D. Resting lymphocytes (B cell, CD4+ T cell, CD8+ T cell)E. B and C

148. Which of the following components of the adaptive immune system causes lysis of virally infected cells and the release of cytokines?

A. Activated B cell (plasma cell)B. CD4+ activated T cellC. CD8+ cytotoxic T cell (CTL)D. Resting lymphocytes (B cell, CD4+ T cell, CD8+ T cell)E. B and C

149. Antigen presenting cells (APCs) include macrophages and dendritic cells, which are found in lymphoid tissues and one other organ. These are critical in the uptake and presentation of antigen to T cells. In what organ APCs also found?

A. LiverB. KidneyC. SkinD. BrainE. Gallbladder

150. Which of the following is NOT a major feature (characteristic) of the adaptive immune system?

A. SpecificityB. DiversityC. MemoryD. ImprovementE. Speed

151. How many doses of the Hepatitis B vaccine (5µg HBsAg in 0.5mL intramuscular) are required to reach a protective level of antibody?

A. OneB. TwoC. ThreeD. FourE. Five

31

Page 32: ZSMU Immunology Mcqs for 5th yr MBBS

152. Match Eosinophils with their function:A. Phagocytosis and bacteriocidal mechanisms, antigen presentationB. Lysis of some virally infected cellsC. Killing parasitesD. Release of histamine and other mediatorsE. Phagocytosis and bacteriocidal mechanisms

153. Match Macrophages with their function:A. Phagocytosis and bacteriocidal mechanisms, antigen presentationB. Lysis of some virally infected cellsC. Killing parasitesD. Release of histamine and other mediatorsE. Phagocytosis and bacteriocidal mechanisms

154. Match Tissue mast cells with their function:A. Phagocytosis and bacteriocidal mechanisms, antigen presentationB. Lysis of some virally infected cellsC. Killing parasitesD. Release of histamine and other mediatorsE. Phagocytosis and bacteriocidal mechanisms

155. Match Natural killer cells with their function:A. Phagocytosis and bacteriocidal mechanisms, antigen presentationB. Lysis of some virally infected cellsC. Killing parasitesD. Release of histamine and other mediatorsE. Phagocytosis and bacteriocidal mechanisms

156. Match Neutrophils with their function:A. Phagocytosis and bacteriocidal mechanisms, antigen presentationB. Lysis of some virally infected cellsC. Killing parasitesD. Release of histamine and other mediatorsE. Phagocytosis and bacteriocidal mechanisms

157. T cells are made in the bone marrow and complete their differentiation in the:A. ThyroidB. SpleenC. Bone marrowD. ThymusE. Thalamus

32

Page 33: ZSMU Immunology Mcqs for 5th yr MBBS

158. Which of the following is a transmembrane glycoprotein that serves as a co-receptor for the T cell receptor (TCR), and is also known as a cytotoxic T cell (CTL)?

A. Cluster of differentiation 4 (CD4+)B. Cluster of differentiation 8 (CD8+)C. Plasma cells (activated B cells)D. A and B are correctE. B and C are correct

159. Which of the following produce large amounts of antibodies (Igs) and differentiate upon stimulation from CD4+ cells?

A. Cluster of differentiation 4 (CD4+)B. Cluster of differentiation 8 (CD8+)C. Plasma cells (activated B cells)D. A and B are correctE. B and C are correct

160. Which of the following is a glycoprotein expressed on the surface of T helper cells, regulatory T cells, monocytes, macrophages, and dendritic cells?

A. Cluster of differentiation 4 (CD4+)B. Cluster of differentiation 8 (CD8+)C. Plasma cells (activated B cells)D. A and B are correctE. B and C are correct

161. Proliferation of cells with matching receptors is the following stage of an adaptive (acquired) immune response:

A. Cognitive phaseB. Activation phaseC. Effector phaseD. “Zero” phaseE. There is no correct phase definition

162. Eventual elimination of antigen is the following stage of an adaptive (acquired) immune response:

A. Cognitive phaseB. Activation phaseC. Effector phaseD. “Zero” phaseE. There is no correct phase definition

163. Recognition of antigen is the following stage of an adaptive (acquired) immune response:

A. Cognitive phaseB. Activation phase

33

Page 34: ZSMU Immunology Mcqs for 5th yr MBBS

C. Effector phaseD. “Zero” phaseE. There is no correct phase definition

164. What stage of an adaptive immune response involves secretion of antibody from a large plasma cell with extensive endoplasmic reticulum?

A. Cognitive phaseB. Activation phaseC. Effector phaseD. “Zero” phaseE. There is no correct phase definition

165. Which of the following types of antigen presenting cells (APCs) is critical in uptake and presentation of antigen to T cells?

A. MacrophageB. Dendritic cellC. B cellD. Plasma cellE. There is no correct answer

166. Which of the following types of antigen presenting cells (APCs) has immunoglobulin that functions as a receptor, then the antigen is internalized, degraded, and presented to T cells?

A. MacrophageB. Dendritic cellC. B cellD. Plasma cellE. There is no correct answer

167. Which of the following types of antigen presenting cells (APCs) is specialized for degradation and presentation of particulate antigens to T cells?

A. MacrophageB. Dendritic cellC. B cellD. Plasma cellE. There is no correct answer

168. Which of the following is NOT true regarding the complement system?A. They are serum proteins that form protein cascades, each activated

component activating the next to generate a physiologic responseB. They can bind to bacteria, making holes in their membraneC. They attract phagocytes to both foreign material and self cellsD. Binding of MBLs to a bacterial capsule triggers the complement cascadeE. They help to eliminate immune complexes (antibody-antigen) and prevent

them from damaging the body

34

Page 35: ZSMU Immunology Mcqs for 5th yr MBBS

169. Which of the following key components of the complement pathway can be activated by the lectin, classical, and alternative pathways?

A. C1B. C3C. C5D. C7E. C9

170. What types of molecules recognize antigens bound to MHC:A. B cell receptor (BCR)B. T cell receptor (TCR)C. Major histocompatibility complex (MHC)D. BCRs & TCRsE. BCRs, TCRs, & MHCs

171. What types of molecules can create a soluble antigen receptor (antibody):A. B cell receptor (BCR)B. T cell receptor (TCR)C. Major histocompatibility complex (MHC)D. BCRs & TCRsE. BCRs, TCRs, & MHCs

172. What types of molecules are cluster of genes that encode proteins:A. B cell receptor (BCR)B. T cell receptor (TCR)C. Major histocompatibility complex (MHC)D. BCRs & TCRsE. BCRs, TCRs, & MHCs

173. Which of the following has an immunoglobulin fold?A. B cell receptor (BCR)B. T cell receptor (TCR)C. Major histocompatibility complex (MHC)D. BCRs & TCRsE. BCRs, TCRs, & MHCs

174. The genes encoding which of the following can undergo hypermutation to create receptors that are an even better fit for foreign antigens?

A. B cell receptor (BCR)B. T cell receptor (TCR)C. Major histocompatibility complex (MHC)D. BCRs & TCRsE. BCRs, TCRs, & MHCs

35

Page 36: ZSMU Immunology Mcqs for 5th yr MBBS

175. The genes encoding which of the following are extensively polymorphic (have multiple alleles or forms of the same gene)?

A. B cell receptor (BCR)B. T cell receptor (TCR)C. Major histocompatibility complex (MHC)D. BCRs & TCRsE. BCRs, TCRs, & MHCs

176. What types of cell is class I MHC found on?A. B cellsB. MacrophagesC. Dendritic cellsD. Essentially all cellsE. Mainly on A, B, & C

177. What types of cell is class II MHC found on?A. B cellsB. MacrophagesC. Dendritic cellsD. Essentially all cellsE. Mainly on A, B, & C

178. What is the function of major histocompatibility complex (MHC) molecules?A. Present sugars to T cellsB. Present peptides to T cellsC. Create holes in the membranes of bacteriaD. Lyse foreign antigensE. Phagocytize foreign antigens

179. Which of the following is NOT true about MHC molecules but true about B and T cells?

A. The genes that encode molecules are the most variable genes we know of in the human genome (polygenic)

B. They are extensively polymorphic (existence of multiple alleles or forms of the same gene)

C. Every cell in each individual expresses the same set of moleculesD. Every cell expresses a different molecule created from multiple gene

segments that undergo somatic rearrangementE. Their diversity exists in the population as a whole, not in the individual

180. Which of the following differentiates an antigen from an immunogen?A. An antigen is a foreign moleculeB. An antigen can cause the production of antibodiesC. An antigen does not always elicit an immune responseD. Antigens are usually proteins or polysaccharides

36

Page 37: ZSMU Immunology Mcqs for 5th yr MBBS

E. Antigens are capable of being bound by immunologic receptors

181. Which of the following is NOT true regarding effective immunogens?A. Foreign to the hostB. Fairly large (molecular weight > 6000)C. Chemically complex (e.g. proteins made of many nucleotide bases)D. Requires a carrier-conjugate to cause the generation of antibodiesE. All of the above

182. Which of the following best describes penicillin, a hapten?A. Large in size and can induce an immune response aloneB. Large in size and needs to be coupled to induce an immune responseC. Small in size and can induce an immune response aloneD. Small in size and needs to be coupled to induce an immune responseE. Penicillin usually do not induce immune responce

183. Which of the following is NOT true?A. The term epitope is not synonymous with antigenB. A viral protein may contain a large number of epitopes that are capable of

interacting with many different specific antibodiesC. Immunologic receptors on T cells recognize continuous (linear) epitopesD. Immunologic receptors on T cells recognize discontinuous

(conformational) epitopesE. Antibodies can recognize both continuous and discontinuous epitopes

184. Which of the following is an agent (e.g. aluminum salts, oil-based, virosomes) often used to modify or augment the effects of a vaccine by stimulating the immune system to respond to the vaccine more vigorously, and thus providing increased immunity to aparticular disease?

A. AntigenB. EpitopeC. ImmunogenD. StimulantE. Adjuvant

185. What fraction of serum are the antibodies (immunoglobulins) initially found when using lytical techniques (e.g. electrophoresis)?

A. AlbuminB. Gamma-globulinC. Beta-globulinD. Alpha1-globulinE. Alpha2-globulin

37

Page 38: ZSMU Immunology Mcqs for 5th yr MBBS

186. In the electrophoresis of human serum, which of the following is the most electronegative and thus migrates farthest toward the positive electrode?

A. AlbuminB. Gamma-globulinC. Beta-globulinD. Alpha1-globulinE. Alpha2-globulin

187. Gamma-globulin serum fraction contains predominantly which immunoglobulin?

A. IgAB. IgDC. IgED. IgGE. IgM

188. The aim of monoclonal antibody production is to produce cells that only secrete immunoglobulin directed against the antigen used in immunization. Which of the following hybridoma production steps is NOT correct?

A. Immunize a mouse with antigen of choice then remove the spleen when the mouse is making an antibody response

B. Fuse the immune spleen cells with a myeloma tumor cellC. The cells are cultured in a selective medium allowing fused and non-fused

cells to surviveD. Cells are grown in individual culture plate wells, and culture supernatants

from wells contains growing hybrid cells are screened for presence of desired antibody by an enzyme-linked immunosorbent assay (ELISA)

E. This clone (hybridoma) is an immortal producer of the desired monoclonal antibody

189. Allows for flexibility of immunoglobulin:A. Light chainB. Heavy chainC. Fab regionD. Fc regionE. Hinge region

190. Binds antigen in immunoglobulin:A. Light chainB. Heavy chainC. Fab regionD. Fc regionE. Hinge region

191. Binds to various cellular receptors and to complement in immunoglobulin:

38

Page 39: ZSMU Immunology Mcqs for 5th yr MBBS

A. Light chainB. Heavy chainC. Fab regionD. Fc regionE. Hinge region

192. Only has fragment antigen-binding (Fab) region in immunoglobulin:A. Light chainB. Heavy chainC. Fab regionD. Fc regionE. Hinge region

193. Has an fragment crystallizable (Fc) and Fab region in immunoglobulin:A. Light chainB. Heavy chainC. Fab regionD. Fc regionE. Hinge region

194. Each antibody molecule contains “X” heavy chains and “Y” light chains, where:

A. X=1; Y=1B. X=1; Y=2C. X=2; Y=1D. X=2; Y=2E. X=2; Y=3

195. Which of the immunoglobulin activates the complement system?A. IgGB. IgEC. IgMD. IgG & IgME. IgG & IgA

196. Which of the immunoglobulin is involved in allergic responses?A. IgGB. IgEC. IgMD. IgG & IgME. IgG & IgA

197. Which of the immunoglobulin is predominant in the primary (early) immune response?

A. IgG

39

Page 40: ZSMU Immunology Mcqs for 5th yr MBBS

B. IgEC. IgMD. IgG & IgME. IgG & IgA

198. Which of the immunoglobulin has different subtypes?A. IgGB. IgEC. IgMD. IgG & IgME. IgG & IgA

199. Which of the immunoglobulin can transfer across the placental (maternal protection)?

A. IgGB. IgEC. IgMD. IgG & IgME. IgG & IgA

200. Pepsin cleaves what region of immunoglobulin?A. Heavy chainB. Light chainC. Hinge regionD. A and BE. None of the above

201. Papain cleaves what region of immunoglobulin?A. Heavy chainB. Light chainC. Hinge regionD. A and BE. None of the above

202. IgM is structurally characterized as:A. MonometricB. BimetricC. TrimetricD. TetrametricE. Pentametric

203. Which of the following is the main immunoglobulin in the gut and secretions (saliva, milk, tears) and is important in mucosal immunity?

A. IgAB. IgD

40

Page 41: ZSMU Immunology Mcqs for 5th yr MBBS

C. IgED. IgGE. IgM

204. Which of the following binds to an Fc receptor on mast cells and basophils?A. IgAB. IgDC. IgED. IgGE. IgM

205. Which of the following is chiefly found on the surface of B cells as a receptor molecule and is involved in cell activation?

A. IgAB. IgDC. IgED. IgGE. IgM

206. Which of the following is NOT involved in the antigen-antibody interaction?A. Electrostatic interactions between charged side-chainsB. Hydrophobic interactionsC. Van der Waals forcesD. Hydrogen bondsE. Peptide bonds

207. Which of the following best describes cross-reactivity?A. When one antibody can bind with one antigenB. When one antibody can bind with multiple antigensC. When multiple antibodies can bind with one antigenD. When multiple antibodies can bind with multiple antigensE. A and B are correct

208. Penicillin can form a hapten-carrier conjugate with a self-protein that can then act as an immunogen and generate certain immunoglobulin antibody, and can cross-react with a number of other antibiotics. This can complicate the treatment of bacterial infections in these patients because they are unable to take the antibiotics necessary to combat the infection. What type of immunoglobulin forms these antibodies?

A. IgAB. IgDC. IgED. IgGE. IgM

41

Page 42: ZSMU Immunology Mcqs for 5th yr MBBS

209. Which of the following is used to enumerate and/or separate live cells that express an antigen, sorted by applying an electric charge to the stained cells?

A. ELISA (Enzyme-linked immunosorbent assay)B. Fluorescent antibody (fluorochromes)C. FACS (fluorescence-activated cell sorting)D. Western blotting (immunoblotting)E. ELISA & Western blotting

210. Which of the following is a very sensitive and simple test for antigens, which uses a covalent complex of enzyme linked to antibody, to detect antigen directly or to bind antibody-antigen complex?

A. ELISA (Enzyme-linked immunosorbent assay)B. Fluorescent antibody (fluorochromes)C. FACS (fluorescence-activated cell sorting)D. Western blotting (immunoblotting)E. ELISA & Western blotting

211. Which of the following is used to characterize antigens in complex mixtures biochemically?

A. ELISA (Enzyme-linked immunosorbent assay)B. Fluorescent antibody (fluorochromes)C. FACS (fluorescence-activated cell sorting)D. Western blotting (immunoblotting)E. ELISA & Western blotting

212. Which of the following uses ultraviolet (UV) light for examining specimens?A. ELISA (Enzyme-linked immunosorbent assay)B. Fluorescent antibody (fluorochromes)C. FACS (fluorescence-activated cell sorting)D. Western blotting (immunoblotting)E. ELISA & Western blotting

213. Which of the following would be used as preliminary screening for the presence of antibodies to HIV proteins in a patient's blood sample?

A. ELISA (Enzyme-linked immunosorbent assay)B. Fluorescent antibody (fluorochromes)C. FACS (fluorescence-activated cell sorting)D. Western blotting (immunoblotting)E. ELISA & Western blotting

214. What color light is emitted when antigens are exposed to UV light after being treated with fluorescein isothiocyte (FITC., such as in the test for Treponema pallidum (syphilis) or to dye the Chicago river for a particular holiday (fluorescein)?

A. Blue

42

Page 43: ZSMU Immunology Mcqs for 5th yr MBBS

B. PurpleC. RedD. OrangeE. Green

215. Which of the following uses the enzyme horseradish peroxidase (HRP)?A. ELISA (Enzyme-linked immunosorbent assay)B. Fluorescent antibody (fluorochromes)C. FACS (fluorescence-activated cell sorting)D. Western blotting (immunoblotting)E. ELISA & Western blotting

216. Which of the following is used extensively to detect antigens in cells or tissue sections, as well as to screen for auto-antibodies to cell or tissue antigens?

A. ELISA (Enzyme-linked immunosorbent assay)B. Fluorescent antibody (fluorochromes)C. FACS (fluorescence-activated cell sorting)D. Western blotting (immunoblotting)E. ELISA & Western blotting

217. Which of the following uses protein antigens separated by molecular weight using sodium dodecyl sulfate polyacrylamide gel electrophoresis (SDS-PAGE.?

A. ELISA (Enzyme-linked immunosorbent assay)B. Fluorescent antibody (fluorochromes)C. FACS (fluorescence-activated cell sorting)D. Western blotting (immunoblotting)E. ELISA & Western blotting

218. Which of the following is only contained in heavy chains and not in light chains?

A. Leader (L)B. Joining (J)C. Diversity (D)D. Variable (V)E. Constant (C)

219. During the development of B cells, the Ig gene segments are rearranged and brought next to each other to form a contiguous functional gene (somatic recombination). The complex of enzymes involved in somatic recombination in lymphocytes is called:

A. RAG-1 (Recombination-Activating Gene)B. RAG-2 (Recombination-Activating Gene)C. V(D)J-recombinaseD. V(D)J-lyaseE. A & B

43

Page 44: ZSMU Immunology Mcqs for 5th yr MBBS

220. The two types of light chains are:A. Alpha and betaB. Sigma and gamma C. Epsilon and delta D. Omega and zeta E. Lambda and kappa

221. During the final step of synthesis for heavy and light immunoglobulin chains, which of the following peptides is cleaved?

A. Leader (L)B. Joining (J)C. Diversity (D.D. Variable (V)E. Constant (C.

222. Which of the following is NOT true regarding the mechanisms of generating antibody diversity?

A. V, D, and J gene segments are present in multiple copies (germline diversity)

B. VJ and VDJ gene segments can recombine in multiple combinations (combinatorial diversity)

C. Different sequences at the joint lead to greater antibody diversity (junctional diversity)

D. A single combination of light and heavy chainsE. Somatic hypermutation after antigenic stimulation

223. With which immunoglobulin IgD is frequently found on the surface of B cells co-expressed? These two classes are co-expressed not by class switching but by alternative processing of a primary RNA transcript. Both molecules expressed on the single mature B cell have the same binding specificity for antigen.

A. IgA B. IgMC. IgED. IgGE. IgM & IgE

224. If alternative processing uses the first polyadenylation site, then what type of heavy chain mRNA is derived?

A. AlphaB. GammaC. DeltaD. EpsilonE. Mu

44

Page 45: ZSMU Immunology Mcqs for 5th yr MBBS

225. If alternative processing uses the second polyadenylation site, then what type of heavy chain mRNA is derived?

A. AlphaB. GammaC. DeltaD. EpsilonE. Mu

226. In heterozygous individuals who have, for example, inherited two alternative forms of the constant region gene for IgG1 (i.e., IgG1m(1) and IgG1m(2)), which of the following is true regarding the Ig expression by a particular B cell, according to allelic exclusion?

A. It will be of the IgG1m(1) typeB. It will be of the IgG1m(2) typeC. It will contain both typesD. It will contain neither typeE. It will contain only one of the types

227. Protein and nucleic acid sequence data have been obtained for many TCRs with different specificities. Analyses of these sequences suggest the existence of how many hypervariable (hv) regions within the variable region?

A. 1B. 2C. 3D. 4E. 5

228. Where are gamma-delta-T cells generally found in the body (location of TCR rearrangement)?

A. SpleenB. Bone marrowC. ThyroidD. ThymusE. Thalamus

229. Which of the following best describes the location of the delta (#)-chain locus in human T-cell receptors?

A. Chromosome 7B. Chromosome 7 within the alpha-locusC. Chromosome 14D. Chromosome 14 within the beta-locusE. Chromosome 14 within the alpha-locus

230. Which of the following are located in chromosome 7 within human T-cell receptors?

45

Page 46: ZSMU Immunology Mcqs for 5th yr MBBS

A. Alpha-locusB. Beta-locusC. Gamma-locusD. A & BE. B & C

231. Which of the following TCR genetic chains contain D-segments, similar to immunoglobulin heavy chains?

A. Alpha; BetaB. Delta; GammaC. Beta; DeltaD. Gamma; AlphaE. Alpha; Delta

232. Which of the following TCR genetic chains contains V and J segments, similar to genes for immunoglobulin kappa and lambda light chains?

A. Alpha; BetaB. Delta; GammaC. Beta; DeltaD. Gamma; AlphaE. Alpha; Delta

233. Deficiencies in which of recombination activating genes lead to autosomal-recessive severe combined immunodeficiency disease (SCID)?

A. RAG-1B. RAG-2C. RAG-1 or RAG-2D. Both RAG-1 and RAG-2E. Other RAG type

234. If both RAG-1 and RAG-2 mutations occur, which of the following is true?A. T cells will be absent but B and NK cells will be presentB. T and B cells will be absent but NK cells will be presentC. B cells will be absent but T and NK cells will be presentD. B and NK cells will be present but T cells will be absentE. B, T, and NK cells will be absent

235. If a B cell successfully rearranges the first heavy chain locus, it inhibits the rearrangement of the other heavy chain on the other chromosome. What is this called?

A. Affinity maturationB. Allelic exclusionC. Class switchingD. Somatic cell hybridizationE. Somatic hypermutation

46

Page 47: ZSMU Immunology Mcqs for 5th yr MBBS

236. Allelic exclusion is a process by which the protein from only one allele is expressed while other allele(s) are silenced. Allelic exclusion occurs (generally) in which of the following?

A. B cells and TCR beta chains, but not in alpha chainsB. B cells and TCR alpha chains, but not in beta chainsC. TCR beta chains, but not in alpha chains or B cellsD. TCR alpha chains, but not in beta chains or B cellsE. B cells, TCR beta chains, and TCR alpha chains

237. A T-cell receptor complex is comprised of the TCR, CD3 molecules, and which of the following CD3 chains?

A. Delta chainB. Gamma chainC. Epsilon chains (two chains)D. Zeta chains (two chains)E. Kappa chain

238. Which of the following molecules is necessary (not accessory) for antigen recognition and T cell activation?

A. CD2B. CD3C. CD4D. CD8E. CD28

239. Co-receptor molecule involved in signal transduction is: A. TCRB. CD3C. CD4 & CD8D. CD11a (LFA-1)E. LMP/TAP

240. Adhesion molecule is:A. TCRB. CD3C. CD4 & CD8D. CD11a (LFA-1)E. LMP/TAP

241. Antigen recognition molecule is:A. TCRB. CD3C. CD4 & CD8D. CD11a (LFA-1)

47

Page 48: ZSMU Immunology Mcqs for 5th yr MBBS

E. LMP/TAP

242. The molecule involved in signal transduction on different T cells is:A. TCRB. CD3C. CD4 & CD8D. CD11a (LFA-1)E. LMP/TAP

243. Genes involved in antigen processing are:A. TCRB. CD3C. CD4 & CD8D. CD11a (LFA-1)E. LMP/TAP

244. Which of the following is the cellular receptor for HIV attachments to T cells?A. CD2B. CD3C. CD4D. CD8E. LFA-1

245. CD4 and CD8 enhance the response of specific T cells, both by stabilizing the TCR-peptide-MHC complex and by bringing what type of protein kinase into the proximity of the cytoplasmic tails, thereby facilitating signal transduction and cell activation?

A. SerineB. ThreonineC. Aspartic acidD. HistidineE. Tyrosine

246. Class III MHC genes encode for which of the following? A. CD4+ T cellsB. CD8+ T cellsC. Compliment moleculesD. Cytokine moleculesE. C & D

247. MHC class I has three genes (HLA-A, HLA-B, & HLA-C). Which of the following is the correct order from most alleles (most polymorphic) to least alleles?

A. A > B > CB. A > C > B

48

Page 49: ZSMU Immunology Mcqs for 5th yr MBBS

C. B > A > CD. B > C > AE. C > A > B

248. How many genes does MHC class II have?A. 2 (HLA-DP alpha/beta)B. 2 (HLA-DQ alpha/beta)C. 4 (HLA-DP alpha/beta, HLA-DQ alpha/beta)D. 4 (HLA-DQ alpha/beta, HLA-DR alpha/beta)E. 6 (HLA-DP alpha/beta, HLA-DQ alpha/beta, HLA-DR alpha/beta)

249. Which of the following MHC class II genes has the most alleles (most polymorphic)?

A. DP-alphaB. DP-betaC. DQ-alphaD. DQ-betaE. DR-beta

250. Which of the following MHC class II genes has the lease alleles?A. DP-alphaB. DP-betaC. DQ-alphaD. DQ-betaE. DR-alpha

251. Because of a lack of genetic recombination in the MHC, blocks of alleles (haplotypes) are inherited together, and they are identical in families. Which of the following is the likely result of this?

A. Family members have identical combinations of HLA alleles, thus tissue transplantation is simple

B. Family members have identical combinations of HLA alleles, thus tissue transplantation is difficult

C. No family members have identical combinations of HLA alleles, thus tissue transplantation is simple

D. No family members have identical combinations of HLA alleles, thus tissue transplantation is difficult

E. All answers are partially correct

252. Which of the following is NOT a structural chain in class I MHC?A. beta-1B. alpha-1C. alpha-2D. alpha-3E. beta-microglobin

49

Page 50: ZSMU Immunology Mcqs for 5th yr MBBS

253. Which of the following is NOT a structural chain in class II MHC?A. alpha-1B. alpha-2C. alpha-3D. beta-1E. beta-2

254. Certain MHC haplotypes are associated with autoimmune disease. Match ankylosing spondylitis with the appropriate HLA type:

A. B27B. DR2C. DR3D. DR4E. DQ2

255. Certain MHC haplotypes are associated with autoimmune disease. Match insulin-dependent diabetes mellitus with the appropriate HLA type:

A. B27B. DR2C. DR3D. DR4E. DQ2

256. Certain MHC haplotypes are associated with autoimmune disease. Match pemphigus vulgaris with the appropriate HLA type:

A. B27B. DR2C. DR3D. DR4E. DQ2

257. Certain MHC haplotypes are associated with autoimmune disease. Match systemic lupus erythematosus with the appropriate HLA type:

A. B27B. DR2C. DR3D. DR4E. DQ2

258. Certain MHC haplotypes are associated with autoimmune disease. Match good pasture’s syndrome with the appropriate HLA type:

A. B27B. DR2C. DR3

50

Page 51: ZSMU Immunology Mcqs for 5th yr MBBS

D. DR4E. DQ2

259. Certain MHC haplotypes are associated with autoimmune disease. Match multiple sclerosis with the appropriate HLA type:

A. B27B. DR2C. DR3D. DR4E. DQ2

260. Certain MHC haplotypes are associated with autoimmune disease. Match rheumatoid arthritis with the appropriate HLA type:

A. B27B. DR2C. DR3D. DR4E. DQ2

261. Which of the following describes where class I MHC is found and not where class II MHC is found?

A. B cellsB. Dendritic cellsC. MacrophagesD. Antigen presenting cells (A, B, & C)E. All nucleated cells

262. Were occurs synthesis of MHC class II and invariant chain (li) in the processing pathway for extracellular antigens?

A. CytosolB. Golgi apparatusC. Endoplasmic reticulumD. RibosomesE. Lysosomes

263. Mutations in TAP-1 or TAP-2 may alter the function of the heterodimer TAP. Which of the following is common for patients with TAP mutations?

A. Human immuno-deficiency virus (HIV) infectionB. Acquired immune deficiency syndrome (AIDS)C. Upper respiratory tract infectionsD. Coagulation disorders (hemophilia)E. Systemic inflammatory response syndrome (SIRS, sepsis)

264. Certain strains of adenovirus express a protein that inhibits the transcription of class I MHC molecules. What is a consequence of this?

51

Page 52: ZSMU Immunology Mcqs for 5th yr MBBS

A. Reduced likelihood that peptides will be expressed at the cell surfaceB. Fewer peptides are available to bind to class I MHCC. Reduced number of class I MHC molecules available to display peptides to

CD8+ lymphocytesD. A and BE. B and C

265. A protein of herpes simplex virus (HSV) binds to TAP and inhibits peptide transport into the endoplasmic reticulum. What is a consequence of this?

A. Reduced likelihood that peptides will be expressed at the cell surfaceB. Fewer peptides are available to bind to class I MHCC. Reduced number of class I MHC molecules available to display peptides to

CD8+ lymphocytesD. A and BE. B and C

266. Bacteria such as Mycobacterium tuberculosis have acquired the capacity to inhibit phagosome-lysosome fusion. What is a consequence of this?

A. Reduced likelihood that peptides will be expressed at the cell surfaceB. Fewer peptides are available to bind to class I MHCC. Reduced number of class I MHC molecules available to display peptides to

CD8+ lymphocytesD. A and BE. B and C

267. The structure of the B-cell receptor contains the invariant proteins Ig-delta and Ig-beta, which are linked via certain bonds and are essential for signal generation across the B-cell surface membrane and for surface expression of immunoglobulin receptors (mIgs). What types of bonds?

A. Covalent bondingB. Hydrophobic interactionsC. Sulfide bondingD. Hydrogen bondingE. Peptide bonding

268. Immunoreceptor tyrosine-based activation motifs (ITAMs) are essential for signal transduction in which cells?

A. B cellsB. T cellsC. B cells or T cellsD. B cells and T cellsE. Neither B or T cells

269. Regarding the structure of the T-cell receptor, which of the following is NOT an invariant CD3 protein chain?

52

Page 53: ZSMU Immunology Mcqs for 5th yr MBBS

A. Zeta chainB. Epsilon chainC. Gamma chainD. Delta chainE. Alpha chain

270. Which of the following is the correct order for a lymphocyte to be activated?A. Recognition of antigen, activation of tyrosine kinases, clustering of

receptors, phosphorylationB. Recognition of antigen, clustering of receptors, activation of tyrosine

kinases, phosphorylationC. Recognition of antigen, activation of tyrosine kinases, phosphorylation,

clustering of receptorsD. Recognition of antigen, activation of tyrosine kinases, clustering of

receptors, phosphorylationE. Recognition of antigen, clustering of receptors, phosphorylation, activation

of tyrosine kinases

271. The B cell co-receptor can co-cluster with the BCR and increase the efficiency of signaling. Which of the following proteins does NOT make up the co-receptor?

A. CD19B. CD20C. CD21D. CD81E. CD18

272. B cell co-receptor phosphorylation sites include Ig-delta/Ig-beta and which of the following?

A. CD19B. CD20C. CD21D. CD81E. CD18

273. Presence of the TCR co-receptor molecules (CD4 or CD8) has been estimated to reduce the number of MHC-peptide complexes required to trigger a T-cell response by about:

A. 5-foldB. 10-foldC. 50-foldD. 100-foldE. 1000-fold

53

Page 54: ZSMU Immunology Mcqs for 5th yr MBBS

274. Protein tyrosine kinases (PTKs) play an important role in lymphocyte function and activation. A mutation in the genes encoding the enzyme ZAP-70 would lead to which of the following?

A. Human immuno-deficiency virus (HIV) infectionB. Acquired immune deficiency syndrome (AIDS)C. Severe combined immunodeficiency (SCID) syndromeD. Systemic inflammatory response syndrome (SIRS, sepsis)E. X-linked agammaglobulinemia

275. A mutation in the genes encoding the enzyme Bruton's tyrosine kinase (Btk) would lead to which of the following?

A. Human immuno-deficiency virus (HIV) infectionB. Acquired immune deficiency syndrome (AIDS)C. Severe combined immunodeficiency (SCID) syndromeD. Systemic inflammatory response syndrome (SIRS, sepsis)E. X-linked agammaglobulinemia

276. What is the major site for hematopoiesis, where all blood cells are formed?A. SpleenB. Bone marrowC. LiverD. ThymusE. Kidneys

277. Which of the following are cells with some lineage commitment and little or no self-renewal capacity?

A. Hematopoietic stem cells (HSCs)B. Progenitor (precursor) cellsC. Mature cellsD. A and B are correctE. B and C are correct

278. Which of the following are lineage restricted, morphologically identifiable, and express differentiation markers?

A. Hematopoietic stem cells (HSCs)B. Progenitor (precursor) cellsC. Mature cellsD. A and B are correctE. B and C are correct

279. Which of the following are pluripotential, self-renewing CD34+ cells?A. Hematopoietic stem cells (HSCs)B. Progenitor (precursor) cellsC. Mature cellsD. A and B are correct

54

Page 55: ZSMU Immunology Mcqs for 5th yr MBBS

E. B and C are correct

280. Which of the following is NOT a cell that progenitors ultimately differentiation into?

A. B cellsB. T cellsC. ErythrocytesD. Hematopoietic stem cells (HSCs)E. Granulocytes

281. Cytokine interleukin-7 (IL-7) is a hematopoietic growth factor secreted by the stromal cells (macrophages and adipocytes) of the red marrow and thymus. What lymphoid cells proliferation it capable of stimulating?

A. Hematopoietic stem cells (HSCs)B. Progenitor (precursor) cellsC. Mature cellsD. B cellsE. T cells

282. To lyse certain virally infected cells and some tumor cells – this is role of:A. B cellsB. T cellsC. NK cellsD. NeutrophilsE. Mast cells

283. Phagocytose, bacteriocidal activity, antibody-dependent cell-mediated cytotoxicity – these are functions of:

A. B cellsB. T cellsC. NK cellsD. NeutrophilsE. Monocytes/Macrophages

284. Important role in allergic response; heparin & histamine production – are features of:

A. B cellsB. T cellsC. NK cellsD. NeutrophilsE. Mast cells

285. Major source of antigen-specific protection against viral infection:A. B cellsB. T cells

55

Page 56: ZSMU Immunology Mcqs for 5th yr MBBS

C. NK cellsD. NeutrophilsE. Mast cells

286. Combat certain parasitic infections (e.g. worms); hydrolytic enzymes – are features of:

A. B cellsB. T cellsC. NK cellsD. EosinophilsE. Mast cells

287. These cells are critical in antigen-capture and uptake in peripheral tissues:A. B cellsB. T cellsC. Dendritic cellsD. NeutrophilsE. Mast cells

288. These cells produce antibody and express immunoglobulin:A. B cellsB. T cellsC. NK cellsD. NeutrophilsE. Mast cells

290. These cells have phagocytic and cytotoxic activities and produce chemotactic factors:

A. B cellsB. T cellsC. NK cellsD. NeutrophilsE. Mast cells

291. The mutation on the X chromosome responsible for X-SCID is in the gene for a subunit of several cytokine receptors. Which of the following is true for boys affected with X-SCID at birth?

A. Non-functional B cells, T cells, & NK cellsB. Non-functional B cells & T cells, and no NK cellsC. Non-functional B cells, and no T cells or NK cellsD. Non-functional T cells & NK cells, and no B cellsE. No B cells, T cells, or NK cells

292. Granulocyte colony-stimulating factor (G-CSF) causes an increase in the production of which of the following in the bone marrow?

56

Page 57: ZSMU Immunology Mcqs for 5th yr MBBS

A. NeutrophilsB. Mast cellsC. EosinophilsD. Monocytes/MacrophagesE. Dendritic cells

293. Macrophage colony-stimulating factor (M-CSF) causes an increase in the production of which of the following?

A. NeutrophilsB. Mast cellsC. EosinophilsD. Monocytes/MacrophagesE. Dendritic cells

294. Granulocyte-macrophage colony-stimulating factor (GM-CSF) causes an increase in the production of which of the following?

A. NeutrophilsB. Mast cellsC. EosinophilsD. Monocytes/MacrophagesE. Dendritic cells

295. In the embryonic human, the primary lymphoid organs are initially in the yolk sac, then in the fetal spleen and liver, and finally in the:

A. Spleen and kidneysB. Spleen and thymusC. Bone marrow and spleenD. Kidneys and bone marrowE. Bone marrow and thymus

296. In the adult, the primary lymphoid organs, whose cells expand clonally, are the bone marrow and thymus and secondary lymphoid organs, which create effector cells, as well as mucosa associated lymphoid tissue (MALT) lining the respiratory, GI, and reproductive tracts, are the following:

A. Thymus and spleenB. Spleen and lymph nodes C. Spleen and bone marrowD. Bone marrow and lymph nodesE. Lymph nodes and thymus

297. The thymus is the primary site of T-cell development. Which of the following is the correct order arranged from the area with the earliest progenitor cells to the area with the most mature T lymphocytes?

A. Medulla then cortex then subcapsular zoneB. Medulla then subcapsular zone then cortex

57

Page 58: ZSMU Immunology Mcqs for 5th yr MBBS

C. Subcapsular zone then medulla then cortexD. Subcapsular zone then cortex then medullaE. Cortex then medulla then subcapsular zone

298. Which of the following is NOT a major area where lymph nodes can be found?

A. CervicalB. AxillaryC. MesentericD. InguinalE. Cardiac

299. Which type of cells contains medulla of the lymph node?A. B cellsB. CD4+ T cellsC. B cells, T cells, macrophagesD. CD8+ T cellsE. B cells and CD8+ T cells

300. Which type of cells contains paracortex of the lymph node?A. B cellsB. CD4+ T cellsC. B cells, T cells, macrophagesD. CD8+ T cellsE. B cells and CD8+ T cells

301. Which type of cells contains cortex of the lymph node?A. B cellsB. CD4+ T cellsC. B cells, T cells, macrophagesD. CD8+ T cellsE. B cells and CD8+ T cells

302. Which of the following locations has cell that transport the antigens by transcytosis into sub-epithelial tissues (e.g. lamina propriA. where they encounter lymphocytes?

A. Tonsils (nasopharyngeal-associated lymphoid tissue, NALT)B. Adenoids (nasopharyngeal-associated lymphoid tissue, NALT)C. Peyer’s patches (gut-associated lymphoid tissue, GALT)D. Axilla (lymph node center)E. Thymus (T cell development)

303. Regarding mucosa-associated lymphoid tissue (MALT), which of the following is secreted by B cells across the epithelium?

A. IgA

58

Page 59: ZSMU Immunology Mcqs for 5th yr MBBS

B. IgDC. IgED. IgGE. IgM

304. Intraepithelial lymphocytes (IEL) act to protect the host against viral and bacterial pathogens encountered in the gut and secrete cytokines. Their predominant phenotype is:

A. alpha beta T cellsB. gamma delta T cellsC. B cellsD. MacrophagesE. Dendritic cells

305. Which type of cells are trafficked via cell adhesion molecules (CAMs), which are a part of the immunoglobulin superfamily?

A. Naive T lymphocytesB. Effector (memory) T lymphocytesC. Both naive T lymphocytes and effector (memory) T lymphocytesD. Either naive T lymphocytes or effector (memory) T lymphocytesE. Neither naive T lymphocytes nor effector (memory) T lymphocytes

306. Which of the following is the correct order of lymphocyte extravasation?A. Lymphocyte activation then adhesion to endothelium then arrested

adhesion then transmigration/chemostaxisB. Arrested adhesion then lymphocyte activation then adhesion to

endothelium then transmigration/chemostaxisC. Arrested adhesion then adhesion to endothelium then lymphocyte

activation then arrested adhesion then transmigration/chemostaxisD. Adhesion to endothelium then arrested adhesion then lymphocyte

activation then transmigration/chemostaxisE. Adhesion to endothelium then lymphocyte activation then arrested

adhesion then transmigration/chemostaxis

307. Lymphocyte passage (diapedesis or transmigration) occurs between adjacent endothelial cells then into the tissues.at the:

A. Gap junctionB. Tight junction (zonula occludens)C. Adherens junction (zonula adherens)D. Desmosome (macular adherens)E. Gap and Adherens junction

308. During B cell development, negative selection occurs in an attempt to ensure the antigen receptor:

A. Can bind with a specific antigen

59

Page 60: ZSMU Immunology Mcqs for 5th yr MBBS

B. Cannot bind with certain foreign antigensC. Can bind to self-antigensD. Cannot bind to self-antigensE. Can bind both to foreign and self antigens

309. Which of the following has the surface marker CD34 instead of the surface marker CD19, during B cell development?

A. Stem cellB. Pro-B cellC. Early pre-B cellD. Late pre-B cellE. Immature cell

310. During B cell development, when is there rearrangement in the heavy (H) chain of the immunoblobulin gene started?

A. Pro-B cell stageB. Early pre-B cell stageC. Late pre-B cell stageD. Immature cell stageE. Stem cell

311. During B cell development, when is there rearrangement in the light (L) chain of the immunoblobulin gene started?

A. Pro-B cell stageB. Early pre-B cell stageC. Late pre-B cell stageD. Immature cell stageE. Stem cell

312. During B cell development, at what stage can cytoplasmic' chains being to be found?

A. Pro-B cell stageB. Early pre-B cell stageC. Late pre-B cell stageD. Immature cell stageE. Stem cell

313. During B cell development, at what stage is membrane bound IgM found?A. Pro-B cell stageB. Early pre-B cell stageC. Late pre-B cell stageD. Immature cell stageE. Stem cell

60

Page 61: ZSMU Immunology Mcqs for 5th yr MBBS

314. Which of the following best describes the role of the pre-B cell receptor (pre BCR)?

A. Signal transduction leading to cell proliferationB. Signal transduction leading to cell apoptosisC. Prevention of self-recognition and thus cell proliferationD. Acquisition of self-recognition and thus cell apoptosisE. Binding with cell adhesion molecules (CAMs)

315. At what stage of B cell development are auto-reactive cells removed?A. Stem cellB. Pro-B cell stageC. Early pre-B cell stageD. Late pre-B cell stageE. Immature cell stage

316. Allelic exclusion creates monospecific B-cell receptors (BCRs). The result is these cells each secrete:

A. Different antibodies with distinct antigenic specificitiesB. Similar antibodies with distinct antigenic specificitiesC. Different antibodies with the same antigenic specificityD. Similar antibodies with the same antigenic specificityE. None of the above

317. What surface immunoglobulins are present once an immature B cell becomes a mature (peripheral) B cell?

A. IgDB. IgMC. IgAD. IgA & IgME. IgM & IgD

318. What are peripheral cells called if they have never come in contact with antigen?

A. Mature cellB. Immature cellC. Naive cellD. Plasma cellE. Stem cell

319. Tolerance refers to the ability of B cells to:A. Tolerate T cellsB. Tolerate other B cellsC. Tolerate self antigensD. Tolerate foreign antigensE. Tolerate changes in pH, body temperature, and other homeostatic changes

61

Page 62: ZSMU Immunology Mcqs for 5th yr MBBS

320. Where does affinity maturation occur?A. Paracortex of lymphoid tissueB. Cortex of lymphoid tissueC. Germinal center of lymphoid tissueD. Follicular dendritic cellsE. Plasma cells

321. Which of the following is the correct order of developmental stages for T cells?

A. Double-positive (DP) then double-negative (DN) then single-positive (SP) B. Double-positive (DP) then single-positive (SP) then double-negative (DN) C. Double-negative (DN) then double-positive (DP) then single-positive (SP) D. Double-negative (DN) then single-positive (SP) then double-positive (DP)E. Single-positive (SP) then Double-negative (DN) then double-positive (DP)

322. In which of the following developmental stages of T cells are surface molecules CD4 and CD8 NOT expressed?

A. Double-negative (DN) cellB. Double-positive (DP) cellC. Single-positive (SP) cellD. Mature T cellE. All of the above

323. Where are single-positive (SP) cells found?A. Bone marrowB. Thymus subcapsular zoneC. Thymus cortexD. Thymus medullaE. Periphery

324. Where are double-negative (DN) cells found?A. Bone marrowB. Thymus subcapsular zoneC. Thymus cortexD. Thymus medullaE. Periphery

325. Where are double-positive (DP) cells found?A. Bone marrowB. Thymus subcapsular zoneC. Thymus cortexD. Thymus medullaE. Periphery

62

Page 63: ZSMU Immunology Mcqs for 5th yr MBBS

326. At what stage does selection occur and where in the thymus does it occur?A. Double-negative (DN); CortexB. Double-positive (DP); CortexC. Single-positive (SP); CortexD. Double-negative (DN); Subcapsular zoneE. Double-positive (DP); Subcapsular zone

327. Approximately how many cells survive the selection process?A. 95%B. 75%C. 50%D. 25%E. 5%

328. Which of the following begins at the double-positive stage and not the double-negative stage?

A. Alpha-chain rearrangementB. Beta-chain rearrangementC. Gamma-chain rearrangementD. Delta-chain rearrangementE. Zeta-chain rearrangement

329. At what stage are rearranged T cell receptors expressed on the cell surface?A. Double-negativeB. Double-positiveC. Single-positiveD. Mature naive T cellE. Single-negative

330. Where does a mature T cell encounter antigen for the first time?A. Freely in the blood streamB. Freely in a primary lymphoid organC. Freely in a secondary lymphoid organD. On an antigen presenting cell (APC) in a primary lymphoid organE. On an antigen presenting cell (APC) in a secondary lymphoid organ

331. Patients with MHC class II (or class I) antigen deficiency would exhibit which of the following?

A. Human immuno-deficiency virus (HIV) infectionB. Acquired immune deficiency syndrome (AIDS)C. Persistent bacterial and viral infectionsD. Coagulation disorders (hemophilia)E. Systemic inflammatory response syndrome (SIRS, sepsis)

63

Page 64: ZSMU Immunology Mcqs for 5th yr MBBS

332. The gamma-delta T cell acts as a part of the first line of defense, recognizing microbial invaders. They appear to recognize commonly occurring microbial pathogens. Where can gamma-delta T cells be found?

A. Liver and kidneyB. Bone marrow and spleenC. Skin and gut mucosaD. Respiratory tract and bone marrowE. Brain and spinal cord

333. Antigens and lymphoid cells are delivered to the secondary lymphoid tissues (spleen, lymph nodes) via:

A. alpha-beta T cellsB. gamma-delta T cellsC. B cellsD. Macrophages E. Dendritic cells

334. Cytokines produced by TH1 (microbial infection response) are:A. IFN-gamma and tumor necrosis factor-betaB. IL-4 and IL-5C. IFN-gamma and IL-4D. Tumor necrosis factor-beta and IL-5E. IFN-gamma and IL-5

335. Cytokines produced by TH2 (parasitic infection response and allergy response) are:

A. IL-4 and IL-5B. IFN-gamma and tumor necrosis factor-betaC. Tumor necrosis factor-beta and IL-5D. IFN-gamma and IL-4E. Tumor necrosis factor-beta and IL-4

336. Cellular interactions between B and T cells involve costimulatory activation. There is a physical interaction between CD40 on the B cell and:

A. CD28on the T cellB. CD4 on the T cellC. CD154 on the T cellD. CD3 on the T cellE. CD80 on the T cell

337. Activation of the gene for IL-2, and eventually TH proliferation, is causes by a critical signal (along with TCR signal transduction) from CD80 on the B cell and:

A. CD40 on the T cellB. CD3 on the T cell

64

Page 65: ZSMU Immunology Mcqs for 5th yr MBBS

C. CD154 on the T cellD. CD28 on the T cellE. CD4 on the T cell

338. What is the immunologic consequence of mutation in CD40L (CD154)?A. Leprosy and immunity to protozoal infectionsB. Acquired immune deficiency syndrome (AIDS)C. X-linked hyper-IgM syndromeD. Systemic inflammatory response syndrome (SIRS, sepsis)E. X-linked agammaglobulinemia

339. Which of the following is required for a CD8+ cell to differentiate into an effector cytotoxic T cell (CTL)? Is this a physical interaction?

A. Recognition of antigen; NoB. Costimulatory signals from CD4+ TH cells; NoC. Recognition of antigen; YesD. Costimulatory signals from CD4+ TH cells; YesE. A and B

340. Which of the following is NOT true when comparing primary immune response to subsequent (secondary) immune response?

A. Primary response takes 5-10 daysB. Secondary response takes 1-3 daysC. Primary response has IgM as the major antibody classD. Secondary response has IgH (IgA or IgE. as the major antibody classE. Primary response has a high affinity for antigen

341. Which of the following is a low affinity antibody?A. IgAB. IgEC. IgHD. IgME. IgD

342. Which of the following best describes the affinity of antibody secreted for effector B cells?

A. HighB. LowC. Increases during responseD. Decreases during responseE. None of the above

343. Which of the following is NOT true regarding apoptosis of target cells?A. Apoptosis is triggered by Fas-Fas interactions and the activation of

Caspase

65

Page 66: ZSMU Immunology Mcqs for 5th yr MBBS

B. The critical cascade step is the activation of Caspase-activatable DNase (CAD)

C. bcl genes are apoptotic genes meant to shorten memory cell survivalD. Apoptosis of memory cells would lead to decreased secondary responseE. Lack of apoptosis would lead to long term survival of memory cells

344. Which of the following is a function of the adaptive immune system and NOT the innate immune system?

A. Distinguishes self from non-selfB. Has preformed or rapidly formed componentsC. Responds within minutes to infectionD. Has no specificity and responds to a range of pathogensE. Uses pattern-recognition molecules

345. Which of the following is a function of the innate immune system and NOT the adaptive immune system?

A. Has possibility of up to 10^18 different receptors B. Recognizes conformational structures or short peptides bound to MHCC. Has immunological memoryD. Frequently malfunctions and may cause autoimmunityE. Uses germline genes to produce collectins

346. Match the innate immune component with the name and location. Low pH:A. Langerhans cells in skinB. Upper respiratory tractC. GI tractD. Dead skin keratinocytesE. Living skin keratinocytes

347. Match the innate immune component with the name and location. Surfactants, pathogen binding collectins:

A. Langerhans cells in skinB. Upper respiratory tractC. GI tractD. Dead skin keratinocytesE. Lower respiratory tract

348. Match the innate immune component with the name and location. Initially prevent organism penetration:

A. Langerhans cells in skinB. Upper respiratory tractC. GI tractD. Dead skin keratinocytesE. Living skin keratinocytes

66

Page 67: ZSMU Immunology Mcqs for 5th yr MBBS

349. Match the innate immune component with the name and location. Mucociliary escalator:

A. Langerhans cells in skinB. Upper respiratory tractC. GI tractD. Dead skin keratinocytesE. Living skin keratinocytes

350. Match the innate immune component with the name and location. Migrate and present antigen to T cells:

A. Langerhans cells in skinB. Upper respiratory tractC. GI tractD. Dead skin keratinocytesE. Living skin keratinocytes

351. Match the innate immune component with the name and location. Secrete cytokines if damaged:

A. Langerhans cells in skinB. Upper respiratory tractC. GI tractD. Dead skin keratinocytesE. Living skin keratinocytes

352. What type of cells produce type I interferons (IFN-alpha and IFN-beta)?A. MonocytesB. Myeloid dendritic cells (mDC)C. Plasmacytoid dendritic cells (pDC)D. Plasma cellsE. Goblet cells

353. Which of the following is NOT true regarding interferon?A. Interferon prevents infection spreading from cell to cellB. NK cells are activated by interferon and lyse infected cellsC. Antigen-presenting cells are inactivatedD. Interferon is viral specific, attacking viral protensE. Stimulation of activity of TAP (transporter-associated with antigen

presentation) occurs

354. In the lectin complement pathway, mannan-binding lecting (MBL) indirectly activated which of the following components?

A. C1B. C2C. C3D. C4

67

Page 68: ZSMU Immunology Mcqs for 5th yr MBBS

E. C2 & C4

355. In the classical complement pathway, which of the following components is the initiating protein(s)?

A. C1B. C2C. C3D. C4E. C2 & C4

356. Which of the following component molecules is activated in the classical pathway after binding to an Fc?

A. C1B. C2C. C3D. C4E. C2 & C4

357. In the alternative complement pathway, this component molecule undergoes spontaneous activation, which creates a challenge for organ xenotransplantation.

A. C1B. C2C. C3D. C4E. C2 & C4

358. Which complement pathway is activated by (and requires) antibodies, with IgM being particularly good at C1 binding?

A. LectinB. ClassicalC. AlternativeD. B or CE. None of the above

359. Anaphylatoxins are chemotaxins that stimulate phagocytosis and degranulation. Which complement components is/are mainly involved?

A. C2 & C4B. C3C. C5D. C3 & C5E. C5-C9

360. Opsonization is the process by which bacteria and other cells are made available for phagocytosis and involves opsonin or IgG. Which complement components is/are mainly involved?

68

Page 69: ZSMU Immunology Mcqs for 5th yr MBBS

A. C2 & C4B. C3C. C5D. C3 & C5E. C5-C9

361. The membrane attack complex (MAC. uses C3 to activate the final part of the complement pathway. It is important in defense against Neisseria. Which of the following components is/are inserted into the plasma membrane of the target cell, allowing for free passage of water/solute and thus killing the cell?

A. C5 & C6B. C7C. C8D. C9E. C7-C9

362. Which of the following is NOT a major function of complement activation?A. OpsonizationB. B-cell stimulationC. T-cell stimulationD. Immune complex clearanceE. None of the above

363. To prevent inadvertent complement activation, eight inhibitors exist. Which of the following is an inhibitor preventing activation of C2 and C4, and prevents C3 activation by the alternative pathway?

A. C1 inhibitorB. C2 inhibitorC. C3 inhibitorD. C4 inhibitorE. C2 & C4 inhibitor

364. A deficiency in complement inhibitors could lead to which of the following?A. MeningitisB. LeprosyC. X-linked hyper-IgM syndromeD. Hereditary angiodemaE. X-linked agammaglobulinemia

365. Pus formed at the site of infection is largely composed of dead:A. MacrophagesB. NeutrophilsC. EosinophilsD. BasophilsE. Lymphocytes

69

Page 70: ZSMU Immunology Mcqs for 5th yr MBBS

366. Which of the following is NOT true of neutrophils compared with macrophages?

A. Rapid increase in production during acute responseB. Is short-lived (dies after phagocytosis)C. Has a single mature formD. Found in healthy tissuesE. Rapidly forms pus

367. Follows concentration gradient of chemokines to the site of infection:A. DiapedesisB. ChemotaxisC. G-CSFD. ChemokinesE. None of the above

368. Uses integrin molecules to squeeze between endothelial cells:A. DiapedesisB. ChemotaxisC. G-CSFD. ChemokinesE. None of the above

369. Makes integrins more “sticky”:A. DiapedesisB. ChemotaxisC. G-CSFD. ChemokinesE. None of the above

370. Increases marrow production of neutrophils:A. DiapedesisB. ChemotaxisC. G-CSFD. ChemokinesE. None of the above

371. Binds sugars on bacteria:A. FcB. CD14 & toll-likeC. ComplementD. Chemokine & cytokineE. C-lectin

372. Mediate adherence and prime for phagocytosis:

70

Page 71: ZSMU Immunology Mcqs for 5th yr MBBS

A. FcB. CD14 & toll-likeC. ComplementD. Chemokine & cytokineE. C-lectin

373. Bind lipopolysaccharide on pathogens and dying cells:A. FcB. CD14 & toll-likeC. ComplementD. Chemokine & cytokineE. C-lectin

374. Bind opsonized pathogens on immune complexes:A. FcB. CD14 & toll-likeC. ComplementD. Chemokine & cytokineE. C-lectin

375. Bind Ig-coated pathogens:A. FcB. CD14 & toll-likeC. ComplementD. Chemokine & cytokineE. C-lectin

376. Match the respiratory burst enzyme with the resulting molecule. NADPH oxidase:

A. HOClB. Nitric oxideC. H2O2D. A and BE. B and C

377. Match the respiratory burst enzyme with the resulting molecule. Myeloperoxidase:

A. HOClB. Nitric oxideC. H2O2D. A and BE. B and C

378. Match the respiratory burst enzyme with the resulting molecule. Nitric oxide synthetase:

71

Page 72: ZSMU Immunology Mcqs for 5th yr MBBS

A. HOClB. Nitric oxideC. H2O2D. A and BE. B and C

379. A 4-year-old boy presents with chronic granulomatous disease (CGD., confirmed with a nitro blue tetrazolium (NBT) test. Which of the following pathogens could this child still mount a defense against?

A. StaphylococcusB. EnterobacteriaC. AspergillusD. StreptococcusE. Pseudomonas

380. What is the primary response seen in acute-phase responses to infection?A. Decreased blood pHB. Decreased metabolic pHC. Increased body temperatureD. Increased heart rateE. Decreased blood pressure

381. Which of the following is a chemoattractant that attracts neutrophils to the site of infection?

A. IL-1B. IL-6C. IL-7D. IL-12E. TNF

382. Which of the following alerts the adaptive immune system to the presence of infection?

A. IL-1B. IL-6C. IL-8D. IL-12E. TNF

383. Which of the following are secreted by macrophages after they have recognizedpathogens using pattern recognition molecules (induction phasE.?

A. IL-1B. IL-6C. IL-7D. TNF

72

Page 73: ZSMU Immunology Mcqs for 5th yr MBBS

E. A, B, & D

384. Which of the following are early clinical signs of (septic) shock?A. Hypertension and bradycardiaB. Hypotension and bradycardiaC. Hypertension and tachycardiaD. Hypotension and tachycardiaE. None of the above

385. Parasitic worm eggs are resistant to low pH and proteolytic digestion in the stomach. Adult worms living inside the lower gut are protected from many of the components of the immune response. Mast cells respond to worms. What immunoglobulin activates these mast cells, leading to degranulation?

A. IgAB. IgDC. IgED. IgGE. IgM

386. Increases vascular permeability:A. Trypase & ChemotrypsinB. HistamineC. Cytokines/IL-4D. Cytokines/TNF E. MMP-1

387. Increase mucus secretion and smooth muscle contraction:A. Trypase & ChemotrypsinB. HistamineC. Cytokines/IL-4D. Cytokines/TNF E. MMP-1

388. Enhance diapedesis and stimulate TH2 responses:A. Trypase & ChemotrypsinB. HistamineC. Cytokines/IL-4D. Cytokines/TNF E. MMP-1

389. Activates TH2 and stimulates eosinophil production:A. Trypase & ChemotrypsinB. HistamineC. Cytokines/IL-4D. Cytokines/TNF

73

Page 74: ZSMU Immunology Mcqs for 5th yr MBBS

E. MMP-1

390. Which of the following would NOT act initially during mast cell (or basophil) activation?

A. TrypaseB. ChemotrypsinC. HistamineD. ProstaglandinsE. Leukotrienes

391. Which of the following is NOT true regarding eosinophils?A. They are specifically recruited to tissues during some types of

inflammationB. Their granules contain particularly toxic substancesC. Their production is stimulated by IL-3 and IL-5D. They are recruited to parasite infected sites by chemokine eotaxinE. They are a major acidic and anionic protein

392. Antibody-dependent cellular cytotoxicity (ADCC) is the process by which natural killer cells destroy infected cells, identified by what immunoglobulin on the surface?

A. IgAB. IgDC. IgED. IgGE. IgM

393. Natural killer cells are thought to fill the gap between initial innate response and adaptive immune response. They evolved to detect low levels of MHC caused by the diseases of Herpes virus and which of follows?

A. Staph bacteriaB. Helminths (worms)C. Tumor cellsD. Influenza virusE. Gonorrhea bacteria

394. Natural killer cell receptors NKG2/CD94 and killer inhibitory receptors allow for the killing of antibodies in the presence of:

A. MHC class IB. MHC class IIC. Both MHC class I and IID. Either MHC class I or IIE. Neither MHC class I nor II

74

Page 75: ZSMU Immunology Mcqs for 5th yr MBBS

395. Match the description with the cytotoxic T cell (CTL) killing mechanism. Potent inducer of apoptosis is:

A. PerforinB. GranzymeC. Fas ligandD. DecorinE. None of the above

396. Match the description with the cytotoxic T cell (CTL) killing mechanism. Degrade host cell proteins, activate caspase enzyme system:

A. PerforinB. GranzymeC. Fas ligandD. DecorinE. None of the above

397. Match the description with the cytotoxic T cell (CTL) killing mechanism. Forms a pore in the target cell membrane (like MAC):

A. PerforinB. GranzymeC. Fas ligandD. DecorinE. None of the above

398. Which of the following is NOT true about live vaccines?A. They were the first vaccines to be discoveredB. They are currently the least effective vaccinesC. They replicate and thus deliver sustained doses of antigenD. They deliver antigenic peptides to MHC class IE. They replicate at the infection site, focusing immune response

399. Which of the following live vaccines is recommended for children but not adults?

A. PoliovirusB. Measles, mumps, rubella (MMR)C. VaricellaD. MeningococcalE. Influenza

400. Which of the following live vaccines is recommended for adults but not children?

A. PoliovirusB. Measles, mumps, rubella (MMR)C. VaricellaD. Meningococcal

75

Page 76: ZSMU Immunology Mcqs for 5th yr MBBS

E. Influenza

401. Killed vaccines are theoretically much safer than live vaccines, but cannot replicate in hosts and cannot enter intracellular antigen presenting pathways. Which of the following killed vaccines is recommended for children but not adults?

A. Hepatitis AB. InfluenzaC. Inactivated poliovirusD. All of the aboveE. None of the above

402. Subunit vaccines contains components from pathogens, which are inactivated and referred to as Toxoids. These vaccines are generally of low immunogenicity and may need something to work effectively. What are they need?

A. CytokinesB. IgEC. B & T cellsD. AdjuvantsE. Antigens

403. Which of the following subunit vaccines is recommended for children but not adults?

A. Pneumonococcal (PCV)B. Haemophilus influenzae type BC. Hepatitis BD. Diptheria, tetanus, pertussisE. None of the above

404. DNA vaccines (tested in micE. use the gene for the immunogenic protein and coat it with what element, which is injected into cells and results in antibody production?

A. TinB. ZincC. CopperD. GoldE. Silver

405. Immunostimulatory complexes (ISCOMS) can be used for mucosal vaccines and induce widespread mucosal immunity in the gut and respiratory tract. What cell response can be promoted with these vaccines?

A. B cellB. Helper T cellC. Cytotoxic T cellD. Eosinophil

76

Page 77: ZSMU Immunology Mcqs for 5th yr MBBS

E. Macrophate

406. Which of the following is NOT true regarding live viral vectored vaccines?A. They require strong adjuvantsB. They are economical to produceC. They can be delivered through needle-free methodsD. They induce a broad and long-lasting immune responseE. They induce both cellular and humoral immunity

407. Which of the following diseases occurs with the absence of a thymus?A. DiGeorge’s syndromeB. Severe combined immunodeficiency disease (SCID)C. Chronic granulomatous disease (CGD)D. Bare lymphocyte syndrome (BLS)E. Wiskott Aldrich syndrome (WAS)

408. Which of the following diseases affect neutrophils?A. DiGeorge’s syndromeB. Severe combined immunodeficiency disease (SCID)C. Chronic granulomatous disease (CGD)D. Bare lymphocyte syndrome (BLS)E. Wiskott Aldrich syndrome (WAS)

409. Which of the following diseases affects mature T cells?A. DiGeorge’s syndromeB. Severe combined immunodeficiency disease (SCID)C. Chronic granulomatous disease (CGD)D. Bare lymphocyte syndrome (BLS)E. Wiskott-Aldrich syndrome (WAS)

410. A child presents with a maculopapular rash on the extremities and trunk. Petechiae is found on the trunk and mucous membranes. Lab tests show a Neisseria infection and the physician is concerned about meningococcemia. Which of the following is the most likely?

A. C3 deficiencyB. C5-C9 deficiencyC. Leukocyte adhesion deficiency (LAD)D. Hyper IgM syndrome (HIM)E. Selective IgA deficiency

411. A patient in their early 20s presents with recurrent bacterial infections. Lab work shows B cells in the periphery as well as low immunoglobulin levels. History shows immunoglobulin levels have decreased with age. Which of the following is the most likely?

A. Myeloperoxidase deficiency

77

Page 78: ZSMU Immunology Mcqs for 5th yr MBBS

B. DiGeorge’s syndromeC. X-linked agammaglobulinemiaD. Wiskott-Aldrich syndrome (WAS)E. Common variable immunodeficiency

412. A child presents with recurrent infections with bacteria, fungi, and viruses. The resident asks the attending physician about IL-2R$ chain mutations, as the patient is a male. The attending physician makes a diagnosis and begins writing orders for bone marrow transplantation and possible gene therapy. Which of the following does the patient have?

A. Ataxia telangiectasiaB. Wiscott-Aldrich syndrome (WAS)C. Hereditary angioedemaD. Severe combined immunodeficiency disease (SCID)E. C3 deficiency

413. A young child presents with cervical adenitis and gingivostomatitis. History reveals recurrent pyogenic infections. Tests are done to determine leukocytosis and the presence of cell adhesion molecules. Which of the following is the most likely?

A. C3 deficiencyB. Wiscott-Aldrich syndrome (WAS)C. Leukocyte adhesion deficiency (LAD)D. Hyper IgM syndrome (HIM)E. Selective IgA deficiency

414. A patient presents in the winter months with swollen lips. The patient says they forgot their lip balm. Uncontrolled activation of the classical complement pathway is responsible for the swelling. Records show the patient has a C1inh deficiency. Which of the following does this patient have?

A. C5-C9 deficiencyB. Glucose-6-phosphate dehydrogenase deficiencyC. Hereditary angioedemaD. Severe combined immunodeficiency disease (SCID)E. C3 deficiency

415. An infant presents with recurrent bacterial infections and partial albinism. Lab work reveals giant granules and neutropenia. Which of the following is the most likely?

A. DiGeorge’s syndromeB. Chediak-Higashi disorderC. Chronic granulomatous disease (CGD)D. Bare lymphocyte syndrome (BLS)E. Myeloperoxidase deficiency

78

Page 79: ZSMU Immunology Mcqs for 5th yr MBBS

416. A male patient presents with severe and recurrent bacterial infections. Lab work confirms the patient is only infected with certain bacteria due to a mutation in a gene for NADPH oxidase. Which of the following is the most likely?

A. DiGeorge’s syndromeB. Chediak-Higashi disorderC. Bare lymphocyte syndrome (BLS)D. Wiskott-Aldrich syndrome (WAS)E. Chronic granulomatous disease (CGD)

417. A 6-year-old presents with the appearance of being intoxicated while walking. Opthlamoscopic findings reveal broken capillaries in the sclera of the eye. Lab work reveals IgA deficiency. Which of the following is the most likely?

A. Ataxia telangiectasiaB. Wiscott-Aldrich syndrome (WAS)C. Hereditary angioedemaD. Myeloperoxidase deficiencyE. C3 deficiency

418. A male infant presents with a triad of symptoms including thrombocytopenia, eczema, and immunodeficiency. Which of the following is the most likely?

A. Transient hypogammaglobulinemia of infancyB. Wiscott-Aldrich syndrome (WAS)C. Hereditary angioedemaD. Selective IgA deficiencyE. Hyper IgM deficiency

419. A young patient presents with severe infections from encapsulated bacteria and collagen vascular disease. Lab work confirms a diagnosis via CH50 and AP50. Which of the following is the most likely?

A. Transient hypogammaglobulinemia of infancyB. Wiscott-Aldrich syndrome (WAS)C. Paroxysmal nocturnal hemoglobinuria (PNH)D. Selective IgA deficiencyE. C3 deficiency

420. An infant presents with recurrent infections with Candida and viruses. Cardiac abnormalities result in the patient requiring heart surgery. During surgery, no thymus is found. Which of the following is the most likely?

A. DiGeorge’s syndromeB. Chediak-Higashi disorderC. Bare lymphocyte syndrome (BLS)D. Wiskott-Aldrich syndrome (WAS)E. Chronic granulomatous disease (CGD)

79

Page 80: ZSMU Immunology Mcqs for 5th yr MBBS

421. A patient presents for a follow-up visit after a diagnosis of systemic lupus erythematosus (SLE.. The physician suspects a misdiagnosis after finding immunу complexes accumulated in the blood, lymph, and tissues. Which of the following is the most likely?

A. C3 deficiencyB. C5-C9 deficiencyC. Early complement deficiencyD. Hyper IgM syndrome (HIM)E. Selective IgA deficiency

422. A patient presents with complaints of dark urine in the morning that clears up partially during the day. Lab work reveals hemolytic anemia, thrombosis in large vessels, and a deficiency in Hematopoiesis. Which of the following is the most likely?

A. Transient hypogammaglobulinemia of infancyB. Wiscott-Aldrich syndrome (WAS)C. Selective IgA deficiencyD. Paroxysmal nocturnal hemoglobinuria (PNH)E. Chediak-Higashi disorder

423. A patient presents with symptoms similar to chronic granulomatous disease (CGD). It is found that there is a problem with generation of NADPH. Which of the following is the most likely?

A. C5-C9 deficiencyB. Glucose-6-phosphate dehydrogenase deficiencyC. Myeloperoxidase deficiencyD. Severe combined immunodeficiency disease (SCID)E. C3 deficiency

424. A patient is being seen for recurrent infections. However, the patient does not seem to think anything special of these infections and there has been no previous diagnosis other than the infections. Lab work reveals abnormalities with hypochlorous acid and H2O2 conversion. Which of the following is the most likely?

A. Ataxia telangiectasiaB. Wiscott-Aldrich syndrome (WAS)C. Hereditary angioedemaD. Myeloperoxidase deficiencyE. C3 deficiency

425. A patient presents at age 6-months with pneumonia and difficulty fighting off infections of encapsulated bacteria and enteroviruses. A mutation in Bruton’s tyrosine kinase (btk) is suspected. Which of the following is the most likely?

A. Myeloperoxidase deficiencyB. DiGeorge’s syndrome

80

Page 81: ZSMU Immunology Mcqs for 5th yr MBBS

C. X-linked agammaglobulinemiaD. Wiskott-Aldrich syndrome (WAS)E. Common variable immunodeficiency

426. A male infant presents with decreased levels of IgA, IgG, and IgE. History reveals recurrent bacterial infections and severe diarrhea. A mutation on CD40L (CD154) on T cells is suspected. Which of the following is the most likely?

A. Bare lymphocyte syndrome (BLS)B. C5-C9 deficiencyC. Early complement deficiencyD. Hyper IgM syndrome (HIM)E. Selective IgA deficiency

427. A patient presents at 3-years-old for a follow-up visit. At age 6-months, the patient had a delayed onset of synthesis of an immunoglobulin. At this visit, the mother reports that the recurrent respiratory infections have stopped due to antibiotics and globulin replacement. Which of the following was the most likely for this patient?

A. Transient hypogammaglobulinemia of infancyB. Wiscott-Aldrich syndrome (WAS)C. Paroxysmal nocturnal hemoglobinuria (PNH)D. Selective IgA deficiencyE. C3 deficiency

428. Which of the following is often asymptomatic to presenting with an increased incidence of respiratory tract infections and is the most common immunodeficiency disease?

A. C3 deficiencyB. C5-C9 deficiencyC. Early complement deficiencyD. Hyper IgM syndrome (HIM)E. Selective IgA deficiency

429. A patient presents with symptoms similar to DiGeorge’s syndrome. It is determined that the patient’s TH cells are unable to develop. Which of the following is the most likey?

A. Leukocyte adhesion deficiency (LAD)B. Chediak-Higashi disorderC. Bare lymphocyte syndrome (BLS)D. Wiskott-Aldrich syndrome (WAS)E. Chronic granulomatous disease (CGD)

430. A 58-year-old white man presents with weight loss, night sweats, and dyspnea. On examination, the patient appears chronically ill and is pale.

81

Page 82: ZSMU Immunology Mcqs for 5th yr MBBS

Laboratory testing reveals leukocytosis, anemia, and throm-bocytopenia. A bone marrow biopsy with aspirate is performed, and a diagnosis of acute myelogenous leukemia is confirmed. In counseling the patient about chemotherapy, you inform him that he is going to be at increased risk for infections and that a major source of infection will be his own gastrointestinal tract. Which of the following statements regarding the innate immune system and the epithelial barrier in the GI tract is false?

A. Lectins found in secretions bind sugars on pathogens and activate the lectin pathway of complement activation

B. Granulocytes marginate in small blood vessels throughout much of the barrier tissues and are available for rapid recruitment to a possible site of infection

C. Mucus itself is a protective barrier that traps organisms and debrisD. Secretions on the epithelial barrier concentrate complement in such a way

that the concentration of complement in secretions is higher than the concentration in plasma

E. Monocytes are present in secretions and in most tissues, where they phagocytose unwanted microbes

431. A 26-year-old female patient has had recurrent infections with pyogenic organisms. She has a follow-up appointment with you today to discuss her options. You remember that complement is a major mechanism by which the innate immune system can act and that certain complement deficiencies can cause disease. Which of the following statements regarding the complement cascade is false?

A. The alternative pathway requires antibodies for initiationB. The three complement pathways are the classical pathway, the alternative

pathway, and the lectin pathwayC. The membrane attack complex (MAC) allows perforation via channel or

pore formation into the foreign membraneD. C3 degradation occurs spontaneously all the time, and C3 fragments bind

to host cells and foreign cells; however, regulatory proteins on host cells protect cells by inactivating such fragments

E. All of the above

432. A 32-year-old African-American woman with systemic lupus erythematosus (SLE) presents to your office for an examination. Her disease course has been complicated by hemolytic anemia, renal disease, synovitis, and rash. Her current regimen consists of low-dose prednisone. During her visit, she says she has done

82

Page 83: ZSMU Immunology Mcqs for 5th yr MBBS

some research on the Internet and wants to know if her SLE is caused by a problem with complement. Which of the following statements regarding complement is false?

A. Almost all inherited complement deficiencies are inherited as autoso-mal dominant traits

B. Immune complexes can lodge in blood vessel walls and activate complement to produce synovitis, vasculitis, dermatitis, and glomerulo-nephritis

C. A deficiency of complement regulatory proteins usually causes excessive activation

D. Deficiencies of early components (e.g., C1q, C1r/C1s, C4, and C2) predispose to SLE, whereas deficiencies of C3, MBL, or MAC components lead to recurrent bacterial infections

E. All of the above

433. A 43-year-old man comes for a routine follow-up. Several months ago, the patient presented for evaluation of weight loss, rash, and iron-deficiency anemia. You diagnosed him as having celiac disease. The patient states he is doing well on his gluten-free diet. He has gained 10 lb since his last visit 2 months ago. Today his anemia is also seen to have improved. You remember that celiac disease results from immune dysregulation, and you are stimulated to learn more about adaptive immunity. Which of the following statements regarding the antigens of the major histocompatibility complex (MHC) is false?

A. There are two structural types of MHC molecules, called class I and class II

B. Clonally determined antigen receptors on B cells recognize and bind to specific peptide-MHC complexes

C. MHC molecules act by binding peptide fragments of antigens that have been processed in specialized antigen-presenting cells

D. Class II antigens are encoded by the HLA-D regionE. All of the above

434. A 22-year-old man presents to establish primary care. He has been healthy most of his life, but he does have type 1 diabetes mellitus, which he reports has been under very good control. He informs you that when last measured, his hemoglobin A1C value was 5.2%. He has no history of retinopathy or neuropathy, and he states that he saw his ophthalmologist 6 weeks ago. The patient has had protein in his urine, and he takes an angiotensin-converting enzyme (ACE) inhibitor. He asks you, “What causes type 1 diabetes?” You explain that the

83

Page 84: ZSMU Immunology Mcqs for 5th yr MBBS

underlying problem is that his body has mistaken its own pancreatic molecules for foreign molecules. Later that day, you decide to read further on adaptive immunity. Which of the following statements regarding antigen processing and presentation is false?

A. Class I molecules are expressed on virtually all tissues and are important in the recognition of virally infected cells

B. Class II molecules are expressed on a limited variety of cells known as antigen-presenting cells

C. MHC molecules first bind peptide fragments after the MHC molecules reach the cell surface

D. Exogenous proteins are taken up by endosomes or lysosomes, where they are catabolized; their peptides are then bound to MHC class II molecules

E. All of the above

435. A 23-year-old primigravida who is known to be Rh-negative is told by her obstetrician that she needs a medication to prevent complications (i.e., erythroblastosis fetalis) of her next pregnancy. She wonders why she should be using this medication. Which of the following immunologic responses is prevented by the use of anti–Rh-positive antibodies (RhoGAM)?

A. Primary immune response to antigenB. Secondary immune response (anamnestic or booster response)C. Somatic hypermutationD. Class switch recombinationE. All of the above

436. A 48-year-old woman with severe rheumatoid arthritis (RA) is advised by a rheumatologist to consider a novel antibody, because her arthritis is not responding to therapy with methotrexate. She asks you about this new medication. Of the following, which is the therapeutic target of approved engineered human monoclonal antibodies in the management of RA?

A. Interleukin-6 (IL-6)B. IL-10C. Tumor necrosis factor–α (TNF-α)D. IL-1E. All of the above

437. A 42-year-old white man presents to primary care clinic complaining of fatigue. Physical examination is significant for splenomegaly. Laboratory data reveal a leukocytosis with 3% blasts and numerous immature cells of the

84

Page 85: ZSMU Immunology Mcqs for 5th yr MBBS

granulocytic lineage, and a low leukocyte alkaline phosphatase (LAP) level. Cytogenetic analysis reveals the Philadelphia chromosome [t(9;22)], and a diagnosis of chronic myelogenous leukemia (CML) is made. Subsequently, the patient seeks evaluation for allogeneic stem cell transplantation. During your discussion with him, you explain the importance of human leukocyte antigen (HLA) matching of donor and recipient to reduce the incidence of graft-versus-host disease. Which cluster of highly polymorphic genes encodes these cell surface markers?

A. The cytokine cluster lociB. The major histocompatibility complex (MHC)C. The cytokine histocompatibility complexD. The chemokine histocompatibility clusterE. All of the above

438. A 72-year-old woman with emphysema presents for evaluation for possible lung transplantation. Laboratory evaluation for cytokine polymorphism of the transforming growth factor (TGF) gene, considered as homozygosity for TGF-α, is associated with graft fibrosis in 93% of lung transplant recipients. TGF-α has two well-studied dimorphic positions within the leader sequence of the gene whose variants are found in concert with one another. What designation is given to variants at polymorphic positions that display this relationship?

A. Hardy-Weinberg equilibriumB. Allelic equilibriumC. Linkage disequilibriumD. Allelic disequilibriumE. All of the above

439. A clinical investigator studying the genetic predisposition of individuals with a family history of diabetes mellitus to develop clinical diabetes discovers a novel genetic polymorphism in a cohort of such patients. Which of the following describes a mutation whose frequency becomes established at more than 1% to 2% of the population?

A. A haplotypeB. An alleleC. A unique polymorphismD. A single nucleotide polymorphismE. None of the above

85

Page 86: ZSMU Immunology Mcqs for 5th yr MBBS

440. A 24-year-old black woman comes to your office complaining of bilateral hand pain, a painful mouth, and a rash on her face that is particularly bothersome when she is exposed to the sun. She has had these symptoms for 6 weeks. Her examination is remarkable for patchy alopecia and multiple mouth ulcers; the musculoskeletal examination is normal. The patient tests positive for both ANA and anti–double-stranded DNA. You make a diagnosis of systemic lupus erythematous, a disease in which autoimmunity is known to play a central role. Which of the following is NOT a possible mechanism of tolerance?

A. Clonal deletion in the thymusB. Failure of T cells bearing low-affinity receptors to recognize antigens in the

peripheryC. Sequestration of an antigen from the immune system as a result of

anatomic barriersD. Acquisition of anergy after ligation of the T cell receptor complex in the

absence of costimulationE. T cells in the thymus with high affinity for a self-antigen undergo positive

selection

441. You are asked to see a 34-year-old pregnant woman in the emergency department who is experiencing shortness of breath. She has pulmonary edema, and an echocardiogram shows mitral stenosis. She is from South America. When asked, she says that many years ago, she had an illness with rash, fever, and joint pain that kept her in bed for a few weeks. On the basis of this history, you make a presumptive diagnosis of rheumatic mitral stenosis. Which of the following constitutes the best immunologic causative mechanism of rheumatic fever?

A. Direct bacterial infection of the heartB. Antistreptocococcal antibodies cross-reacting with myocardial antigensC. Toxins released by group A Streptococcus that cause valvular damageD. Pathogenic autoantibodies directed against the endocardium of heart valvesE. All of the above

442. A 35-year-old woman with a history of asthma and atopic dermatitis presents to your office for follow-up. She was recently hospitalized for community-acquired pneumonia complicated by an acute exacerbation of her asthma. Which of the following statements most accurately describes the T cell response to allergenic pep-tides in an atopic patient?

A. In the TH2 response, T cells form interleukin-4 (IL-4), IL-5, and IL-13, thereby directing the production of allergen-specific antibodies

86

Page 87: ZSMU Immunology Mcqs for 5th yr MBBS

B. In the TH1 response, T cells produce interferon gamma (IFN-γ), thereby inducing T cell differentiation

C. In the THO response, T cells produce IL-12 and IL-18, thereby causing differentiation from THO cells to TH1 cells

D. In the TH response, naive helper T cells differentiate into mature T lymphocytes, producing IgG1 and IgG4 antibodies

E. All of the above

443. A 28-year-old graduate student with a history of chronic allergic rhinitis and asthma presents to your clinic. His symptoms, which are continuous, have been somewhat refractory to the therapies you have tried thus far. He recently ran across a proposed new drug therapy for asthma while reading a scientific journal. The name of this drug is omalizumab, and he asks you to explain how it works. Which of the following responses is the most accurate answer to this patient’s question?

A. Omalizumab is a monoclonal antibody that is directed against the tumor necrosis factor (TNF) receptor; it inhibits the action of TNF

B. Omalizumab is a monoclonal antibody directed against the Fcε portion of IgE; it inhibits activation of mast cells

C. Omalizumab is a monoclonal antibody directed against the IL-5 receptor; it inhibits eosinophil development

D. Omalizumab is a cyclic polypeptide immunosuppressant that suppresses inflammation

E. None of the above

444. An 18-year-old woman is brought to the emergency department after a bee sting. She is flushed and in mild respiratory distress. She is afebrile. Her heart rate is 110 beats/min; her blood pressure is 80/40 mm Hg; and her respiratory rate is 28 breaths/min. As you prepare to initiate supportive therapy and empirical treatment for anaphylaxis, the nurse, who has drawn blood, asks what tests you would like to order. Which of the following serum markers, if elevated, most consistently suggests anaphylaxis as the cause of hypotension?

A. Histamine B. Chymase C. Cathepsin G D. TryptaseE. All of the above

445. A 42-year-old woman presents to your clinic complaining of continuing allergic rhinitis. A biopsy of her nasal mucosa would almost certainly reveal

87

Page 88: ZSMU Immunology Mcqs for 5th yr MBBS

eosinophils. There are several mechanisms that lead to the preferential accumulation of eosinophils, rather than neutrophils, at sites of allergic inflammation. Of the following mediators and receptors, which is specifically involved with eosinophil chemotaxis?

A. Leukotriene C4 (LTC4)B. CCR3 chemokine receptorC. Very late antigen–4 (VLA-4)D. All of the aboveE. None of the above

446. A 28-year-old man presents to your clinic for evaluation of allergies. He has a long history consistent with allergic rhinoconjunctivitis but also experiences urticarial lesions when he eats certain types of food. He also occasionally has back pain from a recent sports injury. His medications include loratadine and low-dose corticosteroids, which were prescribed by his primary care doctor, as well as ibuprofen and a daily baby aspirin. You decide to perform skin testing on the patient. Which of the following interventions should you recommend before performing epicutaneous testing?

A. The patient should discontinue all medications 1 week before testingB. The patient should discontinue loratadine and steroids 3 days before testingC. The patient should discontinue loratadine 1 week before testingD. The patient should discontinue loratadine, steroids, and ibuprofen 1 week

before testingE. None of the above

447. A 35-year-old man comes to your office with symptoms of nasal congestion and itchy eyes and throat. He has been experiencing such symptoms for several years. Symptoms are present throughout the year, and he is able to enjoy outdoor activities without worsening of the symptoms. He owns a cat, which does not sleep in the same room with him. You order allergy skin testing and receive a report indicating a positive response to dust mites and cat dander. Which of the following therapeutic interventions is the most effective for this patient’s symptoms?

A. AntihistaminesB. Removal of the allergen from the patient’s environmentC. Leukotriene receptor antagonistsD. Cromolyn sodiumE. All of the above

88

Page 89: ZSMU Immunology Mcqs for 5th yr MBBS

448. A 20-year-old woman comes to your office in early spring with complaints of nasal congestion, runny nose, and paroxysms of sneezing. She has been experiencing these symptoms for 10 days. She denies having fever, cough, myalgias, or malaise. She states that she typically experiences bouts of similar symptoms in September and October. Her medical history includes mild intermittent asthma since childhood. On examination, she has dark rings under her eyes but no sinus tenderness. The nasal mucosa appears pale and swollen, and there is clear rhinorrhea. Which of the following statements regarding this patient’s condition is false?

A. Nasal smear is likely to show a preponderance of eosinophilsB. Her symptoms are the result of the IgE-mediated release of substances such

as histamine that increase epithelial permeabilityC. Treatment of the condition can result in improvement of coexisting asthma

in certain patientsD. Although daily nasal steroid sprays can alleviate symptoms, they are

generally not recommended because of the risk of rhinitis medicamentosaE. Immunotherapy can be employed in patients whose symptoms persist

despite the avoidance of triggers and the use of pharmacotherapy

449. A 45-year-old man with a history of seasonal allergic rhinitis presents with complaints of itching, tearing, and mild burning of both eyes. He has had these symptoms for several days. He has not had any vision changes or systemic symptoms. He reports that the ocular symptoms began in association with nasal congestion and rhinorrhea, a pattern he has experienced in the past. You suspect that he has allergic conjunctivitis. Which of the following statements regarding the diagnosis and treatment of allergic conjunctivitis is false?

A. Bilateral involvement, although not universal, helps to distinguish the condition from acute infectious conjunctivitis

B. The presence of another atopic disorder such as allergic rhinitis, asthma, or atopic dermatitis (eczema) is present in approximately three fourths of patients with ocular allergy

C. Corticosteroid eyedrops are the most effective treatment and are generally given as first-line agents

D. Patients with viral or bacterial conjunctivitis are more likely to complain of pain and to display matting of the eyelids and purulent ocular discharge

E. All of the above

450. An 18-year-old man comes to clinic complaining of nasal stuffiness, left-sided maxillary tooth pain, and postnasal drip. He has had these symptoms for

89

Page 90: ZSMU Immunology Mcqs for 5th yr MBBS

more than 2 months. After the first 2 weeks of symptoms, he was seen in a walk-in clinic and given a 5-day course of antibiotics, but his symptoms did not improve significantly. He has not had fever or chills but complains that he wakes up with a sore throat on most days; the throat pain tends to get better as the day goes on. On examination, he is afebrile, with mild tenderness to palpation over the left maxilla and left forehead. His posterior oropharynx is slightly erythematous, with yellowish drainage present, but there is no tonsillar exudate. Examination of the nares reveals hyperemic mucosa and mucopurulent discharge. Which of the following statements regarding this patient’s condition is true?

A. Chronic sinusitis can be defined as sinus inflammation that persists for more than 3 weeks

B. Sinus radiographs are the procedure of choice for evaluating patients suspected of having chronic sinusitis

C. It is likely that anaerobic bacteria are the primary pathogens responsible for this patient’s condition

D. Nasal culture has sufficient sensitivity and specificity to guide further antimicrobial therapy

E. In patients with medically resistant chronic sinusitis, further workup for conditions such as cystic fibrosis, structural abnormality, or fungal infection is appropriate

451. A 43-year-old woman comes to your clinic complaining of nonhealing hives. She says that she started having hives 6 weeks ago. The hives are mildly pruritic. When asked, she says that each individual hive lasts for 2 or 3 days. Physical examination reveals multiple urticarial papules that do not blanch on dias-copy. You ask the patient to come back to your clinic after 3 days, and you confirm that some of the lesions are still present. On the basis of this patient’s history and physical examination, what would be the next step in the workup?

A. Administer thyroid function testsB. Perform an abdominal CT scan to rule out an intra-abdominal malignancyC. Check sinus films, hepatitis serology, and stool studies for ova and

parasitesD. Perform a biopsy of one of the lesionsE. A and B

452. A 34-year-old man presents to your clinic complaining of a recurrent, extremely pruritic rash on his trunk and back. The rash started a few months ago. The rash comes and goes; the patient thinks it appears when he exercises or eats spicy foods. Physical examination reveals multiple 2 to 3 mm scattered papular

90

Page 91: ZSMU Immunology Mcqs for 5th yr MBBS

wheals surrounded by large, erythematous flares. Which of the following is a likely diagnosis for this patient?

A. Cholinergic urticaria B. Pressure urticaria C. Idiopathic urticaria D. Aquagenic urticariaE. None of the above

453. While traveling in an airplane, a flight attendant asks you to evaluate a 44-year-old woman who has sudden onset or urticaria, flushing, pruritus, shortness of breath, nausea, and vomiting. You learn that she has a history of allergy to peanuts and that she may have eaten some without knowing it. On physical examination, the patient is alert and is in moderate respiratory distress. Her blood pressure is 90/50 mm Hg, and her heart rate 120 beats/min. She has diffuse inspiratory and expiratory wheezing, and she is experiencing diffuse urticaria. What is the most appropriate treatment for this patient?

A. Administer oxygen and start I.V. steroids and I.V. fluids; the flight can be continued

B. Start an I.V., inject 1 mg of epinephrine I.V., and give I.V. steroids, I.V. fluids, and oxygen; the flight can be continued

C. Administer oxygen and epinephrine subcutaneously or intramuscularly, give I.V. antihistamines and I.V. fluids, start steroids, and ask the pilot to land and transport the patient to an emergency care facility

D. Give oral antihistamines and oral prednisone and continue to watch the patient for further clinical deterioration

E. None of the above

454. A 50-year-old woman is admitted to the hospital with a history of subjective fever of 2 weeks’ duration. The patient underwent mitral valve replacement surgery 5 years ago; in addition, she once experienced an allergic reaction to penicillin, which she describes as a rash that occurred a few minutes after she received a single dose of I.V. penicillin. Physical examination is remarkable for the presence of a diastolic and systolic murmur in the mitral area. Transthoracic echocardiography shows a vegetation in the mitral valve. Blood cultures show penicillin-sensitive viridans streptococci. On the basis of this patient’s history of penicillin allergy, which of the following would be the most appropriate course of action?

A. Start a cephalosporinB. Administer a penicillin skin test before starting antibiotics

91

Page 92: ZSMU Immunology Mcqs for 5th yr MBBS

C. Start a different β-lactam, such as imipenemD. Start vancomycinE. None of the above

455. A 33-year-old man is admitted to the hospital with fever, knee pain, and swelling. Physical examination is remarkable for fever and a swollen, red, painful right knee. Arthrocentesis shows gram-positive cocci in clusters and 150,000 white blood cells. The patient is started on vancomycin. After a few minutes, you are called to see the patient, who is complaining of flushing and back pain. His blood pressure is 90/60 mm Hg, and he has a diffuse erythematous macular rash on his trunk, abdomen, and legs. Which of the following would be the most appropriate course of action for this patient?

A. Administer 0.3 mg of epinephrine I.M., 50 mg of diphenhydramine I.V., and 125 mg of methylprednisolone I.V.

B. Discontinue vancomycin; await culture results and sensitivities before restarting antibiotics

C. Slow down the vancomycin infusion rate and premedicate with diphenhydramine

D. Obtain a vancomycin skin testE. None of the above

456. A 45-year-old man with a history of diabetes and hypertension comes to the emergency department with chest pain. He is found to have a myocardial infarction with ST segment depression. After 4 days in the hospital, the patient has recurrent chest pain; ECG changes are consistent with further ischemia. His cardiologist schedules cardiac catheterization; however, the patient says that 10 years ago, when he had an abdominal CT scan, he had a bad reaction to intravenous contrast.Which of the following would be the most appropriate approach in the management of this patient?

A. Proceed with the catheterization; premedicate with corticosteroids and antihistamines; use nonionic contrast

B. Perform a contrast media radioallergosorbent test (RAST)C. Continue with medical managementD. Obtain a contrast media skin testE. None of the above

457. A 34-year-old woman with AIDS is admitted to the hospital with altered mental status. During workup, she is found to test positive on a Venereal Disease Research Laboratory (VDRL) test and to have elevated levels of white cells in her

92

Page 93: ZSMU Immunology Mcqs for 5th yr MBBS

cerebrospinal fluid. The patient is diagnosed with neurosyphilis. Her sister reports that 15 years ago, the patient had an allergic reaction to penicillin; she describes this reaction as involving lip swelling, hives that appeared all over the patient’s body, shortness of breath, low blood pressure, and diarrhea. These symptoms occurred 10 minutes after receiving a penicillin shot. Which of the following would be the most appropriate course of action for this patient?

A. Premedicate with corticosteroids and antihistamines; start penicillin B. Start ceftriaxoneC. Do not start penicillin; consider erythromycin D. Consult an allergist for desensitizationE. None of the above

458. Allergic reactions to insect stings can be either local or systemic. They result primarily from the stings of insects of the Hymenoptera order, which includes bees, wasps, and imported fire ants. In the United States, at least 40 deaths occur each year as a result of insect stings. Which of the following statements is false?

A. A person who has suffered a number of uneventful stings in the past has no risk of a significant allergic reaction to future stings

B. Although almost 20% of adults demonstrate allergic antibodies to Hymenoptera venom, only 3% of adults and 1% of children suffer from anaphylaxis as the result of being stung

C. Fatalities from systemic allergic reactions are more common in people older than 45 years

D. A person’s risk of anaphylaxis varies in accordance with reactions to previous stings and with results of venom skin tests and radioaller-gosorbent tests (RASTs) for specific IgE antibodies

E. None of the above

459. A patient who in the past suffered from anaphylaxis after a bee sting has recently moved from New York to the southeastern United States. She is concerned about increased exposure to stinging insects in this part of the country and asks your advice. Which one of the following statements might you include in a discussion with this patient regarding the distribution and behavior of various families of Hymenoptera?

A. Africanized honeybees (“killer bees”) are present in the southeastern United States and pose a larger threat in terms of anaphylaxis because the antigen in their venom is unique and is more potent than that found in typical honeybees and bumblebees

93

Page 94: ZSMU Immunology Mcqs for 5th yr MBBS

B. Yellow jackets are relatively docile and tend to stay away from human beings, and they thus pose little threat to this patient

C. Imported fire ants have increasingly become a problem in the southeast but do not tend to cause allergic reactions, because they cause injury only by biting, not stinging

D. Paper wasps, which often build open nests under windowsills or eaves, have the ability to sting multiple times

E. None of the above

460. A mother and her 14-year-old son are in your office. Several months ago, the boy was stung by a wasp. He subsequently developed severe swelling at the site of envenomation; the swelling increased over 24 hours and persisted for several days. He did not, however, develop generalized urticaria, dyspnea, dys-phonia, or weakness. The mother is concerned about the possibility of his having a life-threatening reaction to stings and wants to know what to look for and what tests can be done to determine his risk. Which of the following statements is false?

A. Involvement of the pulmonary and circulatory systems distinguishes a systemic allergic reaction from a severe localized cutaneous reaction

B. RAST must be interpreted in light of the patient’s allergic historybecause venom-specific IgE antibodies may be present in patients who have never demonstrated an allergic reaction to stings

C. RAST is less sensitive than skin testing, and up to 15% to 20% of patients with a documented anaphylactic reaction and positive skin-test results may have undetectable levels of venom-specific IgE antibodies

D. The degree of reaction to a venom skin test (as measured by the size of the wheal and flare) closely correlates with the severity of a patient’s allergic reaction to stings

E. None of the above

461. A 25-year-old woman with a history of eczema presents to the emergency department 2 hours after being stung by a bee while gardening. Initially, swelling occurred at the site of the sting; this was followed by a diffuse urticarial eruption, dyspnea, wheezing, and dizziness. At the triage station, she is awake but somewhat lethargic. She is using accessory muscles to breathe. Her blood pressure is 94/32 mm Hg, and her heart rate is 112 beats/min. Which of the following statements concerning this patient is false?

A. Epinephrine is the initial drug of choice for anaphylactic reactions and may be lifesaving

94

Page 95: ZSMU Immunology Mcqs for 5th yr MBBS

B. If the patient demonstrates initial improvement after treatment, it is safe to discharge her home after observing her for 2 to 4 hours

C. Corticosteroids such as hydrocortisone are appropriate to administer, although their ability to prevent late-phase reactions is debated

D. Before discharge, the patient should be instructed on the use of self-administered epinephrine

E. None of the above

462. A 32-year-old woman whose medical history includes an anaphylactic reaction to a yellow jacket sting would like to know if there are any measures she can take to decrease her risk of anaphylaxis in the event of a future insect sting. You recommend immunotherapy. Which of the following statements best supports your recommendation of immunotherapy for this patient?

A. Children and adults with a history of large local reactions to stings are at relatively high risk for developing anaphylaxis, and venom immunotherapy in these patients is required

B. Although standard venom immunotherapy is generally well tolerated, it is only 40% to 50% effective in completely preventing systemic allergic reactions to stings

C. In a patient who has had a single anaphylactic reaction to a sting and whose skin test is positive, immunotherapy is indicated

D. In a patient receiving immunotherapy, the skin test usually becomes negative within the first 4 to 6 months; failure to do so may indicate a lack of response to the treatment

E. None of the above

463. A 36-year-old man is being evaluated for diarrhea. The patient has a 3-month history of diarrhea, postprandial nausea and vomiting, and weight loss. There is no specific food that he can relate to his symptoms. A complete blood count reveals anemia and eosinophilia. His serum IgE level is increased. Small bowel biopsy reveals eosinophilic infiltration without vasculitis. Which of the following is the most likely diagnosis for this patient?

A. Oral allergy syndromeB. Churg-Strauss syndromeC. Eosinophilic gastroenteropathyD. Immediate gastrointestinal hypersensitivityE. None of the above

95

Page 96: ZSMU Immunology Mcqs for 5th yr MBBS

464. A 3-year-old boy is brought to your office by his mother, who relates that her son was diagnosed as having peanut hypersensitivity 1 year ago. He developed urticaria and nasal congestion after ingestion of peanuts. Since then, he has had two more episodes of hypersensitivity, with similar symptoms. His mother asks about treatment. Which of the following is the most appropriate treatment for this patient?

A. Long-term use of antihistamines B. Immunotherapy C. Elimination diet D. KetotifenE. None of the above

96

Page 97: ZSMU Immunology Mcqs for 5th yr MBBS

Answers

1 E2 D3 C4 E5 B6 D7 A8 D9 B10 D11 C12 D13 A14 D15 D16 D17 B18 A19 D20 D21 D22 D23 B24 D25 D26 A27 D28 D29 A30 C31 B32 B33 B34 D35 D36 E37 D

38 A39 B40 C41 A42 E43 C44 A45 B46 D47 D48 D49 A50 A51 C52 B53 D54 C55 D56 A57 D58 E59 A60 B61 C62 C63 A64 E65 A66 E67 D68 C69 B70 B71 C72 B73 D74 A

75 B76 C77 C78 D79 B80 C81 D82 B83 D84 A85 B86 C87 E88 B89 B90 C91 D92 D93 B94 B95 D96 B97 B98 A99 B100 C101 B102 D103 D104 D105 B106 B107 C108 D109 C110 D111 E

97

Page 98: ZSMU Immunology Mcqs for 5th yr MBBS

112 C113 D114 B115 E116 B117 D118 C119 A120 D121 E122 E123 E124 E125 C126 D127 E128 A129 B130 C131 C132 C133 D134 C135 E136 D137 A138 B139 C140 D141 E142 C143 E144 B145 E146 D147 A148 E149 C

150 E151 C152 C153 A154 D155 B156 E157 D158 B159 C160 A161 B162 C163 A164 C165 D166 C167 A168 C169 B170 B171 A172 C173 D174 A175 C176 D177 E178 B179 D180 C181 D182 D183 D184 E185 B186 A187 D

188 C189 E190 C191 D192 A193 B194 D195 D196 B197 C198 E199 A200 A201 C202 E203 A204 C205 B206 E207 B208 C209 C210 A211 D212 B213 A214 E215 E216 B217 D218 C219 C220 E221 A222 D223 B224 E225 C

98

Page 99: ZSMU Immunology Mcqs for 5th yr MBBS

226 E 227 C 228 D

99

Page 100: ZSMU Immunology Mcqs for 5th yr MBBS

229 E230 E231 C232 D233 C234 B235 B236 A237 D238 B239 B240 D241 A242 C243 E244 C245 E246 E247 C248 E249 E250 E251 D252 A253 C254 A255 E256 D257 C258 B259 B260 D261 E262 C263 C264 C265 B266 A

267 C268 A269 D270 B271 B272 A273 D274 C275 E276 B277 B278 C279 A280 D281 B282 C283 E284 E285 B286 D287 C288 C289 A290 D291 C292 A293 D294 E295 E296 B297 D298 E299 C300 B301 A302 C303 A304 B

305 C306 E307 B308 D309 A310 A311 C312 B313 D314 A315 E316 D317 E318 C319 C320 C321 C322 A323 D324 B325 C326 B327 E328 A329 B330 E331 C332 C333 E334 A335 A336 C337 D338 C339 E340 E341 D342 C

100

Page 101: ZSMU Immunology Mcqs for 5th yr MBBS

343 C344 A345 E346 C347 E348 D349 B350 A351 E352 C353 C354 E355 A356 C357 C358 B359 D360 B361 E362 C363 A364 D365 B366 D367 B368 A369 D370 C371 E372 D373 B374 C375 A376 C377 A378 B379 D380 C

101

Page 102: ZSMU Immunology Mcqs for 5th yr MBBS

381 B382 D383 E384 D385 C386 B387 A388 D389 C390 E391 E392 D393 C394 D395 C396 B397 A398 B399 A400 D401 C402 D403 B404 D405 C406 A407 A408 C

409 D410 B411 E412 D413 C414 C415 B416 E417 A418 B419 E420 A421 C422 D423 B424 D425 C426 D427 A428 E429 C430 D431 A432 A433 B434 C435 B436 C

437 B438 C439 B440 E441 B442 A443 B444 D445 D446 C447 B448 D449 C450 E451 D452 A453 C454 B455 C456 A457 D458 A459 D460 D461 B462 C463 C464 C

102

Page 103: ZSMU Immunology Mcqs for 5th yr MBBS

NOTES

103

Page 104: ZSMU Immunology Mcqs for 5th yr MBBS

104

Page 105: ZSMU Immunology Mcqs for 5th yr MBBS

105


Recommended